MOMMI/BBY

¡Supera tus tareas y exámenes ahora con Quizwiz!

For the first hour after birth, the height of the fundus is at the umbilicus or even slightly above it. a) True b) False

True

Butorphanol tartrate (Stadol) is prescribed for a woman in labor, and the woman asks the nurse about the purpose of the medication. The nurse should make which most appropriate response? 1. "The medication provides pain relief during labor." 2. "The medication will help prevent any nausea and vomiting." 3. "The medication will assist in increasing the contractions." 4. "The medication prevents respiratory depression in the newborn infant."

1 Butorphanol tartrate is an opioid analgesic that provides systemic pain relief during labor. It does not relieve nausea, increase uterine contractions, or prevent respiratory depression in the newborn infant.

The nurse in the newborn nursery is performing admission vital signs on a newborn infant. The nurse notes that the respiratory rate of the newborn is 50 breaths per minute. Which action should the nurse take? 1. Document the findings. 2. Contact the health care provider. 3. Apply an oxygen mask to the newborn infant. 4. Cover the newborn infant with blankets and reassess the respiratory rate in 15 minutes.

1 The normal respiratory rate for a normal newborn is 30 to 60 breaths per minute. On assessment, if the nurse noted a respiratory rate of 50 breaths per minute, the nurse should document these findings because they are normal. Options 2, 3, and 4 are inappropriate or unnecessary nursing actions.

A newborn infant of a mother who has human immunodeficiency virus (HIV) infection is tested for the presence of HIV antibodies. An enzyme-linked immunosorbent assay (ELISA) is performed, and the results are positive. Which is the correct interpretation of these results? 1. Positive for HIV 2. Indicates the presence of maternal infection 3. Indicates that the newborn will develop AIDS later in life 4. Positive for acquired immunodeficiency syndrome (AIDS)

2 A positive antibody test in a child younger than 18 months of age indicates only that the mother is infected because maternal immunoglobulin G antibodies persist in infants for 6 to 9 months and, in some cases, as long as 18 months. A positive ELISA does not indicate true HIV infection or the development of AIDS, nor does it indicate that the newborn will develop AIDS later in life.

Which would be considered a normal finding in a newborn less than 12 hours old? 1. Grunting respirations 2. Heart rate of 190 beats/min 3. Bluish discoloration of the hands and feet 4. A yellow discoloration of the sclera and body

3 Having bluish hands and feet is termed acrocyanosis and is a normal finding in the newborn. Grunting respirations is a sign of possible respiratory distress and the normal newborn heart rate is 100 to 160 beats/min. A yellow discoloration of the sclera and skin indicates jaundice.

The nurse is caring for a post-term, small-for-gestational age (SGA) newborn infant immediately after admission to the nursery. What should the nurse monitor as the priority? 1. Urinary output 2. Total bilirubin levels 3. Blood glucose levels 4. Hemoglobin and hematocrit levels

3 The most common metabolic complication in the SGA newborn infant is hypoglycemia, which can produce central nervous system abnormalities and mental retardation if not corrected immediately. Urinary output, although important, is not the highest-priority action, because the post-term SGA infant is typically dehydrated as a result of placental dysfunction. Hemoglobin and hematocrit levels are monitored because the post-term SGA infant exhibits polycythemia, although this also does not require immediate attention. The polycythemia contributes to increased bilirubin levels, usually beginning on the second day after delivery.

The nurse in the delivery room is performing an initial assessment on a newborn infant. When examining the umbilical cord, the nurse should expect to observe which finding? 1. One artery 2. Two veins 3. Two arteries 4. One artery and one vein

3 The umbilical cord is made up of two arteries to carry blood from the embryo to the chorionic villi and one vein that returns blood to the embryo. There should be no odor noted from the umbilical cord. Options 1, 2, and 4 are incorrect.

The nurse is assessing a newborn infant with a diagnosis of hiatal hernia. Which findings would the nurse most specifically expect to note in the infant? 1. Excessive oral secretions 2. Bowel sounds heard over the chest 3. Hiccups and spitting up after a meal 4. Coughing, wheezing, and short periods of apnea

4 Clinical manifestations associated with hiatal hernia specifically include vomiting, coughing, wheezing, short periods of apnea, and failure to thrive. Excessive oral secretions are a clinical manifestation of esophageal atresia and tracheoesophageal fistula. Bowel sounds heard over the chest is a clinical manifestation associated with congenital diaphragmatic hernia. Hiccups and spitting up after a meal is a clinical manifestation of gastroesophageal reflux.

The nurse administers erythromycin ointment (0.5%) to the eyes of a newborn and the mother asks the nurse why this is performed. Which explanation is best for the nurse to provide about neonatal eye prophylaxis? 1. Protects the newborn's eyes from possible infections acquired while hospitalized. 2. Prevents cataracts in the newborn born to a woman who is susceptible to rubella. 3. Minimizes the spread of microorganisms to the newborn from invasive procedures during labor. 4. Prevents an infection called ophthalmia neonatorum from occurring after delivery in a newborn born to a woman with an untreated gonococcal infection.

4 Erythromycin ophthalmic ointment 0.5% is used as a prophylactic treatment for ophthalmia neonatorum, which is caused by the bacterium Neisseria gonorrhoeae. Preventive treatment of gonorrhea is required by law. Options 1, 2, and 3 are not the purposes for administering this medication to a newborn infant.

The nurse is providing instructions to a new mother regarding cord care for a newborn infant. Which statement, if made by the mother, indicates a need for further instructions? 1. "The cord will fall off in 1 to 2 weeks." 2. "Alcohol may be used to clean the cord." 3. "I should cleanse the cord two or three times a day." 4. "I need to fold the diaper above the cord to prevent infection."

4 The diaper should be folded below the cord to keep urine away from the cord, so a statement by the client that the diaper should be folded above the cord would be incorrect, indicating the need for further instruction. The cord should be kept clean and dry to decrease bacterial growth. Cord care is required until the cord dries up and falls off, between 7 and 14 days after birth. The cord should be cleansed two or three times a day with soap and water or other prescribed agents.

The nurse is preparing to administer an injection of vitamin K to a newborn. Which injection site should the nurse select? 1. The gluteal muscle 2. The lower aspect of the rectus femoris muscle 3. The medial aspect of the upper third of the vastus lateralis muscle 4. The lateral aspect of the middle third of the vastus lateralis muscle

4 The preferred injection site for vitamin K in the newborn infant is the lateral aspect of the middle third of the vastus lateralis muscle in the infant's thigh. This muscle is the preferred injection site because it is free of major blood vessels and nerves and is large enough to absorb the medication.

The mother of a 1-month-old infant is bottle-feeding her infant and asks the nurse about the stomach capacity of an infant. What should the nurse tell the client is the stomach capacity of a 1-month-old infant? 1. 10 to 20 mL 2. 30 to 90 mL 3. 75 to 100 mL 4. 90 to 150 mL

4 The stomach capacity is 10 to 20 mL for a newborn infant, 30 to 60 mL for a 1-week-old infant, 75 to 100 mL for a 2- to 3-week-old infant, and 90 to 150 mL for a 1-month-old.

Bonding between a mother and her infant can be defined how? a) An ongoing process in the year after delivery b) The skin to skin contact that occurs in the delivery room c) A process of developing an attachment and becoming acquainted with each other d) Family growing closer together after the birth of a new baby

A process of developing an attachment and becoming acquainted with each other Correct Explanation: Bonding in the maternal-newborn world is the attachment process that occurs between a mother and her newborn infant. This is how the mother and infant become engaged with each other and is the foundation for the relationship. Because bonding is a process and not a single event, option B is incorrect. The process of bonding is not a year-long process, so option C is incorrect. The family growing closer together after the birth of a new baby is not bonding, so option D is incorrect.

Pain should be assessed regularly in all newborn infants. If the infant is displaying physiologic or behavioral cures indicating pain, measures should be taken to manage the infant's pain. Examples of nonpharmacologic pain management techniques include (select all that apply) a. Swaddling b. Nonnutritive sucking (pacifier) c. Skin-to-skin contact with the mother d. Sucrose e. Acetaminophen

A, B, C, D

As part of Standard Precautions, nurses wear gloves when handling the newborn. The chief reason is a. To protect the baby from infection b. It is part of the Apgar protocol c. To protect the nurse from contamination by the newborn d. Because the nurse has primary responsibility for the baby during the first 2 hours

C

A 25-year-old gravida 2, para 2-0-0-2 gave birth 4 hours ago to a 9-pound, 7-ounce boy after augmentation of labor with Pitocin. She puts on her call light and asks for her nurse right away, stating, "I'm bleeding a lot." The most likely cause of postpartum hemorrhage in this woman is: A. Retained placental fragments B. Unrepaired vaginal lacerations C. Uterine atony D. Puerperal infection

C A. Incorrect: Although retained placental fragments may cause postpartum hemorrhage, this typically would be detected in the first hour after delivery of the placenta and is not the most likely cause of hemorrhage in this woman. B. Incorrect: Although unrepaired vaginal lacerations may cause bleeding, they typically would occur in the period immediately after birth. C. Correct: This woman gave birth to a macrosomic boy after Pitocin augmentation. The most likely cause of bleeding 4 hours after delivery, combined with these risk factors, is uterine atony. D. Incorrect: Puerperal infection can cause subinvolution and subsequent bleeding, but it typically would be detected after 24 hours postpartum. p. 595

Excessive blood loss after childbirth can have several causes; the most common is: A. Vaginal or vulvar hematomas B. Unrepaired lacerations of the vagina or cervix C. Failure of the uterine muscle to contract firmly D. Retained placental fragments

C A. Incorrect: Although these are a possible cause of excessive blood loss, uterine muscle failure (uterine atony) is the most common cause. B. Incorrect: Although this is a possible cause of excessive blood loss, uterine muscle failure (uterine atony) is the most common cause. C. Correct: Uterine atony can best be thwarted by maintaining good uterine tone and preventing bladder distension. D. Incorrect: Although this is a possible cause of excessive blood loss, uterine muscle failure (uterine atony) is the most common cause. p. 595

A nurse is responsible for teaching new parents about the hygienic care of their newborn. The nurse should tell the parents to a. Avoid washing the head for at least 1 week to prevent heat loss. b. Sponge bathe only until the cord has fallen off. c. Cleanse the ears and nose with cotton-tipped swabs, such as Q-tips. d. Water temperature should be at least 38° C.

D

Because a full bladder prevents the uterus from contracting normally, nurses intervene to help the woman empty her bladder spontaneously as soon as possible. If all else fails, the last thing the nurse might try is: A. Pouring water from a squeeze bottle over the woman's perineum B. Placing oil of peppermint in a bedpan under the woman C. Asking the physician to prescribe analgesics D. Inserting a sterile catheter

D A. Incorrect: Pouring water over the perineum may stimulate voiding. It is easy, noninvasive, and should be tried early on. B. Incorrect: The oil of peppermint releases vapors that may relax the necessary muscles. It is easy, noninvasive, and should be tried early on. C. Incorrect: If the woman is anticipating pain from voiding, pain medications may be helpful. Other nonmedical means could be tried first, but medications still come before insertion of a catheter. D. Correct: Invasive procedures usually are the last to be tried, especially with so many other simple and easy methods available (e.g., water, peppermint vapors, pain pills). p. 598

Although circumcision continues to be a controversial procedure, many parents in the United States elect to have this surgery performed on their newborn sons. It is believed that newborns do not feel pain; therefore this is the optimum time for the procedure to be done and no anesthesia is required. Is this statement true or false?

False

The _________________________ test is used to detect the amount of fetal blood in the maternal circulation.

Kleihauer-Betke p. 603

The nurse is discussing infant care as part of the mother-infant's couplet discharge planning. The mother asks the nurse "When will my baby's cord fall off?" The nurse responds, "Your baby's cord should fall off by _______________ after birth."

2 weeks

The nurse weighing a term newborn during the initial newborn assessment determines the infant's weight to be 4325 g. The nurse determines that this infant may be at risk for which complications? Select all that apply. 1. Retinopathy 2. Hypoglycemia 3. Fractured clavicle 4. Hyperbilirubinemia 5. Congenital heart defect 6. Necrotizing enterocolitis

2, 3, 5 Any newborn weighing more than 4000 g at birth is defined as being large for gestational age (LGA). Because of their size, LGA infants are also at risk for hypoglycemia. LGA infants also have a higher incidence of birth injuries (fractured clavicle), asphyxia, and congenital anomalies (heart defect). Retinopathy is a disorder that affects the developing vessels of preterm infants. Hyperbilirubinemia is not an immediate risk related to LGA. Preterm birth is the most prominent risk factor in the development of necrotizing enterocolitis.

The nurse is assessing a newborn who was born to a mother who is addicted to drugs. Which assessment finding would the nurse expect to note during the assessment of this newborn? 1. Lethargy 2. Sleepiness 3. Constant crying 4. Cuddles when being held

3 A newborn of a woman using drugs is irritable. The infant is overloaded easily by sensory stimulation. The infant may cry incessantly and be difficult to console. The infant would hyperextend and posture rather than cuddle when being held.

A newborn is delivered via spontaneous vaginal delivery. On reception of the crying newborn, the nurse's priority is to perform which action? 1. Determine Apgar score. 2. Auscultate the heart rate. 3. Thoroughly dry the newborn. 4. Take the newborn's rectal temperature.

3 An optimal thermal environment is essential to the effective care of a neonate. If a newborn is not thoroughly dried and placed in a warm environment immediately after delivery, cold stress may result. Infants respond to cold stress through an increased need for oxygen and depletion of glucose stores, resulting in an increased respiratory rate and possibly cyanosis. Although auscultating the heart rate is essential in the initial assessment of the newborn, palpating the heart rate via the umbilical cord can be done while drying the infant. Drying the infant should only take a few seconds and auscultating the heart rate can be done immediately afterward. The Apgar score is assessed at 1 and 5 minutes of life. Taking the temperature is not a priority immediately following delivery.

A client in her sixth week postpartum complains of general weakness. The client has stopped taking iron supplements that were prescribed to her during pregnancy. The nurse would assess the client for which of the following? 1. Hypertension 2. Hypovolemia 3. Hyperglycemia 4. Hyperthyroidism

Hypovolemia Correct Explanation: The nurse should assess the client for hypovolemia as the client must have had hemorrhage during birth and puerperium. Additionally, the client also has discontinued iron supplements. Hyperglycemia can be considered if the client has a history of diabetes. Hypertension and hyperthyroidism are not related to discontinuation of iron supplements.

The period encompassing the first 1 to 2 hours after birth often is referred to as the ____________________ stage of labor.

fourth p. 585

After teaching a group of pregnant women about the skin changes that will occur after the birth of their newborn, which statement indicates the need for additional teaching? a) "This line on my belly will go away over time." b) "I can't wait for these stretch marks to disappear after delivery." c) "My nipples won't be so dark after I give birth." d) "I might lose some hair, but it will grow back."

"I can't wait for these stretch marks to disappear after delivery." Correct Explanation: Stretch marks gradually fade to silvery lines but do not disappear completely. As estrogen and progesterone levels decrease, the darkened pigmentation on the abdomen, face, and nipples gradually fades

A new mother is concerned because it is 24 hours after childbirth and her breasts have still not become engorged with breast milk. How should the nurse respond to this concern? a) "You are experiencing lactational amenorrhea. It may be several weeks before your milk comes in." b) "You may have developed mastitis. I'll ask the physician to examine you." c) "I'm sorry to hear that. There are some excellent formulas on the market now, so you will still be able to provide for your infant's nutritional needs." d) "It takes about 3 days after birth for milk to begin forming."

"It takes about 3 days after birth for milk to begin forming." Correct Explanation: The formation of breast milk (lactation) begins in a postpartal woman regardless of her plans for feeding. For the first 2 days after birth, an average woman notices little change in her breasts from the way they were during pregnancy as, since midway through pregnancy, she has been secreting colostrum, a thin, watery, prelactation secretion. On the third day post birth, her breasts become full and feel tense or tender as milk forms within breast ducts and replaces colostrum. There is no need to recommend formula feeding to the mother. Mastitis is inflammation of the lactiferous (milk-producing) glands of the breast; there is no indication that the client has this condition. Lactational amenorrhea is the absence of menstrual flow that occurs in many women during the lactation period.

A woman who gave birth to a healthy newborn 2 months ago comes to the clinic and reports discomfort during sexual intercourse. Which suggestion by the nurse would be most appropriate? a) "You might try using a water-soluble lubricant to ease the discomfort." b) "This is entirely normal, and many women go through it. It just takes time." c) "It takes a while to get your body back to its normal function after having a baby." d) "Try doing Kegel exercises to get your pelvic muscles back in shape."

"You might try using a water-soluble lubricant to ease the discomfort." Correct Explanation: Coital discomfort and localized dryness usually plague most postpartum women until menstruation returns. Water-soluble lubricants can reduce discomfort during intercourse. Although it may take some time for the woman's body to return to its prepregnant state, telling the woman this does not address her concern. Telling her that dyspareunia is normal and that it takes time to resolve also ignores her concern. Kegel exercises are helpful for improving pelvic floor tone but would have no effect on vaginal dryness.

An infant returns to the nursing unit following surgery for a diagnosis of esophageal atresia with tracheoesophageal fistula (TEF). The infant is receiving intravenous fluids and a gastrostomy tube is in place. Following assessment, the nurse positions the infant and performs which action? 1. Elevates the gastrostomy tube 2. Tapes the gastrostomy tube to the bed linens 3. Attaches the gastrostomy tube to low suction 4. Connects the gastrostomy to the feeding pump

1 In the immediate postoperative period, the gastrostomy tube is elevated, allowing gastric contents to pass into the small intestine and air to escape. This promotes comfort and decreases the risk of leakage at the anastomosis. The remaining options are incorrect. Taping the tube to the bed linens presents a risk of accidental removal. Attaching the tube to suction could disrupt the surgical repair site. Feedings are not initiated in the immediate postoperative period.

The postpartum nurse teaches a mother how to give a bath to the newborn infant and observes the mother performing the procedure. Which observation indicates a lack of understanding of the instructions? 1. The mother bathes the newborn infant after a feeding. 2. The mother states that she would gather all supplies before the bath is started. 3. The mother states that she would never leave the newborn infant in the tub of water alone. 4. The mother fills a clean basin or sink with 2 to 3 inches of water and then checks the temperature with her wrist.

1 It is not advisable to bathe a newborn infant after a feeding because handling may cause regurgitation. Because bathing is thought to be relaxing to the infant, bathing before feeding may be the best time. All other options are appropriate interventions in teaching the mother how to bathe a newborn.

The nurse is preparing to bathe a 1-day-old newborn. Which action should the nurse avoid when performing the procedure? 1. Immersing the newborn in water 2. Supporting the newborn's body during the bath 3. Ensuring that the water temperature is warm 4. Ensuring that the water temperature does not exceed 100° F

1 Newborn infants may be immersed in water after the umbilical stump has healed. The infant's body must be supported at all times during the bath. Water should be warm, not hot. A bath thermometer may be used to check the temperature of the water, which should not exceed 100° F. If a thermometer is not available, water that is comfortable when tested on the inside of the wrist or elbow is appropriate.

The nurse in a newborn nursery is monitoring a preterm newborn for respiratory distress syndrome. Which assessment findings would alert the nurse to the possibility of this syndrome? 1. Tachypnea and retractions 2. Acrocyanosis and grunting 3. Hypotension and bradycardia 4. Presence of a barrel chest and acrocyanosis

1 A newborn infant with respiratory distress syndrome may present with clinical signs of cyanosis, tachypnea or apnea, nasal flaring, chest wall retractions, or audible grunts. Acrocyanosis, a bluish discoloration of the hands and feet, is associated with immature peripheral circulation, and is common in the first few hours of life. Options 2, 3, and 4 do not indicate clinical signs of respiratory distress syndrome.

The nurse is preparing to instruct a client in how to bathe a newborn. Which statement should the nurse include in the instruction? 1. "Begin with the eyes and face." 2. "Begin with the feet and work upward." 3. "Do the back side first, and then the front side." 4. "Start with the chest, move to the face, and then finish the rest of the body."

1 Bathing should start at the eyes and face, usually the cleanest area. Next the external ear and the area behind the ears are cleansed. The newborn's neck should be washed because formula, lint, or breast milk will often accumulate in the folds of the neck. Hands and arms are then washed. The newborn's legs are washed next, and the diaper area is washed last.

An initial assessment on a large-for-gestational age (LGA) newborn infant is being done. Which physical assessment technique should the nurse assist in performing to assess for evidence of birth trauma? 1. Palpate the clavicles for a fracture. 2. Auscultate the heart for a cardiac defect. 3. Blanch the skin for evidence of jaundice. 4. Perform Ortolani's maneuver for hip dislocation.

1 Because of the newborn infant's large size, there is an increased risk for shoulder dystocia. This may result in fractured clavicles or brachial plexus palsy or both. Other complications related to birth trauma include facial paralysis, phrenic nerve palsy, depressed skull fractures, hematomas, and bleeding. A cardiac defect would not be related to birth trauma, even though there is an increase in cardiac defects such as transposition of the great vessels in the LGA newborn infant. Jaundice would not be present initially. Hip dislocation is a congenital disorder and is not caused by birth trauma.

The nurse in the newborn nursery is assessing a neonate who was born of a mother addicted to cocaine. Which would the nurse expect to note in the neonate? 1. Tremors 2. Bradycardia 3. Flaccid muscles 4. Extreme lethargy

1 Clinical symptoms at birth in neonates exposed to cocaine in utero include tremors, tachycardia, marked irritability, muscular rigidity, hypertension, and exaggerated startle reflex. These infants are difficult to console and exhibit an inability to respond to voices or environmental stimuli. They are often poor feeders and have episodes of diarrhea.

The nurse is caring for a term newborn. Which assessment finding would alert the nurse to suspect the potential for jaundice in this infant? 1. Presence of a cephalhematoma 2. Infant blood type of O negative 3. Birth weight of 8 pounds 6 ounces 4. A negative direct Coombs' test result

1 Enclosed hemorrhage, such as with cephalhematoma, predisposes the newborn to jaundice by producing an increased bilirubin load as the cephalhematoma resolves and the blood is absorbed into the circulatory system. The classic Rh incompatibility situation involves an Rh-negative mother with an Rh-positive fetus or newborn. The birth weight in option 3 is within the acceptable range for a term newborn and therefore does not contribute to an increased bilirubin level. A negative direct Coombs' test result indicates that no maternal antibodies are present on fetal erythrocytes.

A newborn infant is diagnosed with gastroesophageal reflux (GER), and the infant's mother asks the nurse to explain the diagnosis. On what description should the nurse plan to base the response? 1. Gastric contents regurgitate back into the esophagus. 2. The esophagus terminates before it reaches the stomach. 3. Abdominal contents herniate through an opening of the diaphragm. 4. A portion of the stomach protrudes through the esophageal hiatus of the diaphragm.

1 GER is regurgitation of gastric contents back into the esophagus. Option 2 describes esophageal atresia. Option 3 describes a congenital diaphragmatic hernia. Option 4 describes a hiatal hernia.

On delivery of a newborn, the nurse performs an initial assessment. When should the nurse plan to determine the Apgar score? 1. At 1 minute after birth and 5 minutes after birth 2. Immediately at birth, 3 minutes after birth, and 10 minutes after birth 3. At 1 minute after birth, 5 minutes after birth, and 10 minutes after birth 4. At 1 minute after birth, after the cord is cut, and after the mother delivers the placenta

1 One of the earliest indicators of successful adaptation of the newborn is the Apgar score. This test is performed 1 minute after birth and again 5 minutes after birth.

The mother of a newborn calls the clinic and reports that when cleaning the umbilical cord, she noticed that the cord was moist and that discharge was present. What is the most appropriate nursing instruction for this mother? 1. Bring the infant to the clinic. 2. This is a normal occurrence. 3. Increase the number of times that the cord is cleaned per day. 4. Monitor the cord for another 24 to 48 hours and call the clinic if the discharge continues.

1 Symptoms of umbilical cord infection are moistness, oozing, discharge, and a reddened base around the cord. If symptoms of infection occur, the client should be instructed to notify a health care provider (HCP). If these symptoms occur, antibiotics may be necessary. Options 2, 3, and 4 are inappropriate nursing interventions for the description given in the question.

The nurse is assessing the reflexes of a newborn infant. In eliciting the Moro reflex, the nurse should perform which action? 1. Make a loud, abrupt noise to startle the newborn. 2. Stimulate the ball of the foot of the newborn by firm pressure. 3. Stimulate the perioral cavity of the newborn infant with a finger. 4. Stimulate the pads of the newborn infant's hands by firm pressure.

1 The Moro reflex is elicited by placing the newborn on a flat surface and striking the surface or making a loud, abrupt noise to startle the newborn. The newborn assumes sharp extension and abduction of the arms with the thumbs and forefingers in a C position; this is followed by flexion and adduction to an "embrace" position (legs follow a similar pattern). The Moro reflex is present at birth and is absent by 6 months of age if neurological maturation is not delayed. A persistent response lasting more than 6 months may indicate a neurological abnormality. The rooting reflex is elicited by stimulating the perioral area with the finger. The palmar grasp reflex is elicited by stimulating the palm of the hand by firm pressure, and the plantar grasp reflex is elicited by stimulating the ball of the foot by firm pressure.

A nurse is preparing to care for a newborn who has respiratory distress syndrome. Which initial action should the nurse plan to best facilitate bonding between the newborn and the parents? 1. Encourage the parents to touch their newborn. 2. Identify specific caregiving tasks that may be assumed by the parents. 3. Explain the equipment that is used and how it functions to assist their newborn. 4. Give the parents pamphlets that will help them understand their newborn's condition.

1 The best initial action to begin the attachment process and promote bonding is to encourage the parents to touch their newborn. The parents' initial need is to become acquainted with their newborn. Option 2 may be frightening to the parents because of the condition of the newborn and the unfamiliarity of high-risk newborn care practices. This option will be appropriate later, as the newborn's condition becomes stable. Option 3 is important but is not specific to parent-newborn bonding activities. Option 4 is inappropriate initially. Requiring parents to focus on pamphlets or literature does not enhance the parent-newborn bond.

The nurse is caring for a newborn. Blood samples for serum chemistries are drawn, and the total calcium level is reported as 8.0 mg/dL. How should the nurse interpret this laboratory value? 1. A normal value 2. Lower than normal 3. Higher than normal 4. Requiring health care provider notification

1 Total calcium levels are 7.0 to 12.0 mg/dL in a term infant younger than 1 week and 8.0 to 10.5 mg/dL in a child. Neonatal hypocalcemia is defined as a total serum calcium level of less than 7.0 mg/dL. If a newborn baby's calcium level is abnormal, the nurse should notify the health care provider.

Which are modes of heat loss in the newborn? Select all that apply. 1. Radiation 2. Urination 3. Convection 4. Conduction 5. Evaporation

1, 3, 4, 5 The newborn can lose heat through radiation, convection, conduction, and evaporation. Heat is not lost through urination.

Which would be considered abnormal findings in a newborn less than 12 hours old? Select all that apply. 1. Grunting respirations 2. Presence of vernix caseosa 3. Heart rate of 190 beats/minute 4. Anterior fontanelle measuring 5.0 cm 5. Bluish discoloration of hands and feet 6. A yellow discoloration of the sclera and body

1, 3, 6 Grunting respirations is a sign of possible respiratory distress. The normal newborn heart rate is 100 to 160 beats/minute. The presence of a yellow discoloration could indicate newborn jaundice. Options 2, 4, and 5 are normal findings. The anterior fontanelle should measure 5 cm wide by 2-3 cm long.

The nurse develops a plan of care for a woman with human immunodeficiency virus infection and her newborn. The nurse should include which intervention in the plan of care? 1. Monitoring the newborn's vital signs routinely 2. Maintaining standard precautions at all times while caring for the newborn 3. Initiating referral to evaluate for blindness, deafness, learning problems, or behavioral problems 4. Instructing the breast-feeding mother regarding the treatment of the nipples with nystatin ointment

2 An infant born to a mother infected with human immunodeficiency virus (HIV) must be cared for with strict attention to standard precautions. This prevents the transmission of HIV from the newborn, if infected, to others and prevents transmission of other infectious agents to the possibly immunocompromised newborn. Mothers infected with HIV should not breast-feed. Options 1 and 3 are not associated specifically with the care of a potentially HIV-infected newborn.

The nursery room nurse is assessing a newborn infant who was born to a mother who abuses alcohol. Which assessment finding should the nurse expect to note? 1. Lethargy 2. Irritability 3. Higher-than-normal birth weight 4. A greater-than-normal appetite when feeding

2 Characteristic behaviors of the newborn infant with fetal alcohol syndrome (FAS) are similar to those of the drug-exposed newborn infant. These behaviors include irritability, tremors, poor feeding, and hypersensitivity to stimuli. Newborn infants with FAS are smaller at birth and present with failure to thrive. Head circumference and weight are most affected (smaller head circumference and decreased weight).

The nurse notes hypotonia, irritability, and a poor sucking reflex in a full-term newborn on admission to the nursery. The nurse suspects fetal alcohol syndrome and is aware that which additional sign would be consistent with this syndrome? 1. Length of 19 inches 2. Abnormal palmar creases 3. Birth weight of 6 lb, 14 oz 4. Head circumference appropriate for gestational age

2 Fetal alcohol syndrome is caused by maternal alcohol use during pregnancy. Features of newborns diagnosed with fetal alcohol syndrome include craniofacial abnormalities, intrauterine growth restriction, cardiac abnormalities, abnormal palmar creases, and respiratory distress. Options 1, 3, and 4 are normal assessment findings in the full-term newborn infant.

The postpartum nurse is providing instructions to the mother of a newborn with hyperbilirubinemia who is being breast-fed. The nurse should provide which most appropriate instruction to the mother? 1. Feed the newborn less frequently. 2. Continue to breast-feed every 2 to 4 hours. 3. Switch to bottle-feeding the infant for 2 weeks. 4. Stop breast-feeding and switch to bottle-feeding permanently.

2 Hyperbilirubinemia is an elevated serum bilirubin level. At any serum bilirubin level, the appearance of jaundice during the first day of life indicates a pathological process. Early and frequent feeding hastens the excretion of bilirubin. Breast-feeding should be initiated within 2 hours after birth and every 2 to 4 hours thereafter. The infant should not be fed less frequently. Switching to bottle-feeding for 2 weeks or stopping breast-feeding permanently is unnecessary.

The nurse is preparing to assist in administering neonatal resuscitation with a ventilation bag and mask because the newborn is apneic, gasping, and has a heart rate below 100 beats/min. The nurse should understand that how many ventilations per minute should be delivered to this neonate? 1. 20 to 40 breaths/min 2. 40 to 60 breaths/min 3. 70 to 80 breaths/min 4. 80 to 100 breaths/min

2 If the newborn is apneic or has gasping respirations after stimulation, or the heart rate is below 100 beats/min, positive pressure ventilation by bag and mask can be given. The anesthesia bag used for neonatal resuscitation should have a pressure gauge. Ventilations should be given at a rate of 40 to 60 breaths/min at pressures of 15 to 20 cm H2O.

The nurse is preparing to check the respirations of a newborn who was just delivered. The nurse performs the procedure and should determine that the respiratory rate is normal if which respiratory rate is noted? 1. A respiratory rate of 20 breaths/min 2. A respiratory rate of 40 breaths/min 3. A respiratory rate of 70 breaths/min 4. A respiratory rate of 80 breaths/min

2 Normal respiratory rate varies from 30 to 60 breaths/min when the infant is not crying. Respirations should be counted for 1 full minute to ensure an accurate measurement because the newborn is a periodic breather. Observing and palpating respirations while the infant is quiet promote accurate data collection.

The nurse is performing Apgar scoring for a newborn immediately after birth. The nurse notes that the heart rate is less than 100, respiratory effort is irregular, and muscle tone shows some extremity flexion. The newborn grimaces when suctioned with a bulb syringe, and the skin color indicates some cyanosis of the extremities. The nurse should most appropriately document which Apgar score for the newborn? 1. 3 2. 5 3. 7 4. 10

2 One of the earliest indicators of successful adaptation of the newborn is the Apgar score. Scores range from 0 to 10. Five criteria are used to measure the infant's adaptation. Heart rate: absent = 0; less than 100 = 1; greater than 100 = 2. Respiratory effort: absent = 0; slow or irregular weak cry = 1; good, crying lustily = 2. Muscle tone: limp or hypotonic = 0; some extremity flexion = 1; active, moving, and well flexed = 2. Irritability or reflexes (measured by bulb suctioning): no response = 0; grimace = 1; cough, sneeze, or vigorous cry = 2. Color: cyanotic or pale = 0; acrocyanotic, cyanosis of extremities = 1; pink = 2. Newborn infants with an Apgar score of 5 to 7 often require resuscitative interventions. Scores of less than 5 indicate that the newborn infant is having difficulty adjusting to extrauterine life and requires more vigorous resuscitation.

A 4-day-old newborn is receiving phototherapy at home for a bilirubin level of 14 mg/dL. The nurse should plan to include which instruction in the teaching plan of care during the home visit to the mother of the newborn? 1. Applying lotions to exposed newborn skin 2. Assessing skin integrity and fluid status of the newborn 3. Having minimal contact with the newborn to prevent stimulation 4. Advising the mother to limit the newborn's oral intake during phototherapy

2 Phototherapy is the use of intense fluorescent lights to reduce serum bilirubin levels in the newborn. Assessing skin integrity and fluid status of the newborn infant is an essential component of phototherapy. Lotions are not used to ensure the therapeutic effect of light exposure in subcutaneous tissue. Contact with the newborn infant is important. Adequate oral fluids are essential to prevent dehydration because diarrhea is a common side effect of therapy. In addition, safe care for the newborn infant during phototherapy requires shielding the eyes with a soft eye shield to prevent retinal damage, keeping the newborn's skin exposed except for the wearing of a diaper, and changing the newborn's position frequently.

A nurse is teaching the mother of a newborn infant measures to maintain the infant's health. The nurse identifies which as an example of primary prevention activities for the infant? 1. Selective placement of the infant 2. Periodic well-baby examinations 3. Phenylketonuria (PKU) testing at birth 4. Administration of an antibiotic for an umbilical cord staphylococcal infection

2 Primary prevention activities are actions that are designed to prevent a disease from occurring or to reduce the probability of occurrence of a specific illness. Periodic well-baby examinations focus on health education, nutrition, concerns related to adequate housing, recreation, and genetics. Selective placement of the infant is vague and does not provide any specific information. PKU testing at birth is an example of secondary prevention because it relates to early diagnosis and treatment. Option 4 identifies an actual treatment.

The nurse is performing an assessment of a newborn admitted to the nursery after birth. On assessment of the newborn's head, what should the nurse anticipate to be the most likely finding? 1. A depressed anterior fontanel 2. A soft and flat anterior fontanel 3. An anterior fontanel measuring 1 cm 4. An anterior fontanel measuring 7 cm

2 The anterior fontanel is diamond-shaped and located on the top of the head. It should be soft and flat and may range in size from almost nonexistent to 4 to 5 cm across. It normally closes by 18 to 24 months of age. A depressed fontanel may indicate dehydration.

A client who is positive for human immunodeficiency virus (HIV) delivers a newborn infant. The nurse provides instructions to help the client regarding care of her infant. Which client statement indicates the need for further instruction? 1. "I will be sure to wash my hands before and after bathroom use." 2. "I need to breast-feed, especially for the first 6 weeks postpartum." 3. "Support groups are available to assist me with understanding my diagnosis of HIV." 4. "My newborn infant should be on antiviral medications for the first 6 weeks after delivery."

2 The mode of perinatal transmission of human immunodeficiency virus (HIV) to the fetus or neonate of an HIV-positive woman can occur during the antenatal, intrapartal, or postpartum period. HIV transmission can occur during breast-feeding. HIV-positive clients should be encouraged to bottle-feed their infants per the health care provider's prescription. Frequent hand-washing is encouraged. Support groups and community agencies can be identified to assist the parents with the newborn infant's home care, the impact of the diagnosis of HIV infection, and available financial resources. It is recommended that infants of HIV-positive clients receive antiviral medications for the first 6 weeks of life.

The nurse is checking a newborn's 1-minute Apgar score based on the following assessment. The heart rate is 160 beats/min; he has positive respiratory effort with a vigorous cry; his muscle tone is active and well-flexed; he has a strong gag reflex and cries with stimulus to the soles of his feet; his body is pink, with his hands and feet cyanotic. Which is the newborn's 1-minute Apgar score? 1. 7 2. 9 3. 8 4. 10

2 The newborn has a score of 9 because his heart rate, respiratory effort, muscle tone, and reflex irritability all have a score of 2, with color having a score of 1 because of the acrocyanosis.

The nurse is planning care for a newborn of a mother with diabetes mellitus. What is the priority nursing consideration for this newborn? 1. Developmental delays because of excessive size 2. Maintaining safety because of low blood glucose levels 3. Choking because of impaired suck and swallow reflexes 4. Elevated body temperature because of excess fat and glycogen

2 The newborn of a diabetic mother is at risk for hypoglycemia, so maintaining safety because of low blood glucose levels would be a priority. The newborn would also be at risk for hyperbilirubinemia, respiratory distress, hypocalcemia, and congenital anomalies. Developmental delays, choking, and an elevated body temperature are not expected problems.

The nurse is preparing to listen to the apical heart rate of a newborn. The nurse performs the procedure and should note that the heart rate is normal if which rate is noted? 1. A heart rate of 100 beats/min 2. A heart rate of 140 beats/min 3. A heart rate of 180 beats/min 4. A heart rate of 190 beats/min

2 The normal heart rate in a newborn is 110 to 160 beats/min. The other options are incorrect.

The nurse in a newborn nursery is performing an assessment of an infant. What procedure should the nurse use to measure the infant's head circumference? 1. Wrap the tape measure around the infant's head, and measure just below the eyebrows. 2. Place the tape measure under the infant's head, wrap around the occiput, and measure just above the eyebrows. 3. Place the tape measure under the infant's head at the base of the skull, and wrap around to the front just below the eyes. 4. Place the tape measure at the back of the infant's head, wrap around across the ears, and measure across the infant's mouth.

2 To measure head circumference, the nurse should place the tape measure under the infant's head, wrap the tape around the occiput, and measure just above the eyebrows so that the largest area of the occiput is included. Options 1, 3, and 4 are incorrect methods to measure the head circumference.

Which newborn is most at risk for a brachial plexus injury? 1. A term infant with a history of a forceps-assisted delivery 2. A term infant delivered via primary cesarean section for malpresentation 3. A large for gestational age infant with a history of shoulder dystocia at delivery 4. A 36-week preterm infant delivered vaginally after preterm rupture of membranes

3 Brachial plexus injuries, a fractured clavicle, or a fractured humerus are all possible risks during a delivery of an infant with shoulder dystocia and must be considered during the immediate newborn assessment. Stretching or pulling away of the shoulder from the head may occur during a difficult delivery such as one involving shoulder dystocia. This positioning may cause damage to the upper plexus. Larger infants are more likely to be involved in a delivery in which inadequate space is a concern. In most cases, option 4 would result in an infant of smaller size, so shoulder dystocia would not be a priority risk. Shoulder dystocia does not occur during cesarean section, which eliminates option 2. Option 1 can be eliminated because with a forceps delivery, priority concern is for facial or head injuries more than shoulder, arm, or clavicle injuries.

A nurse is monitoring a newborn infant who has been circumcised. The nurse notes that the infant has a temperature of 100.6° F and that the dressing at the circumcised area is saturated with a foul-smelling drainage. Which is the priority nursing action? 1. Reinforce the dressing. 2. Document the findings. 3. Contact the health care provider. 4. Swab the drainage and send the sample to the laboratory for culture.

3 Complications after circumcision include bleeding, failure to urinate, displacement of the Plastibell, and infection (indicated by a fever and a purulent or foul-smelling drainage). If signs of infection occur, the health care provider is notified. The nurse would change, not reinforce, the dressing; reinforcing the dressing leaves the foul smelling drainage in contact with the surgical site. The nurse would document the findings, but this is not the priority item. The health care provider will prescribe a culture if it is necessary; it is not within the realm of nursing responsibilities to prescribe a diagnostic test.

The nurse assisted with the delivery of a newborn. Which nursing action is most effective in preventing heat loss by evaporation? 1. Warming the crib pad 2. Closing the doors to the room 3. Drying the infant with a warm blanket 4. Turning on the overhead radiant warmer

3 Evaporation of moisture from a wet body dissipates heat along with the moisture. Keeping the newborn dry by drying the wet newborn at birth prevents hypothermia via evaporation. Hypothermia caused by conduction occurs when the newborn is on a cold surface, such as a cold pad or mattress, and heat from the newborn's body is transferred to the colder object (direct contact). Warming the crib pad assists in preventing hypothermia by conduction. Convection occurs as air moves across the newborn's skin from an open door and heat is transferred to the air. Radiation occurs when heat from the newborn radiates to a colder surface (indirect contact).

The nurse in the newborn nursery is preparing to complete an initial assessment on a newborn infant who was just admitted to the nursery. The nurse should place a warm blanket on the examining table to prevent heat loss in the infant caused by which method? 1. Radiation 2. Convection 3. Conduction 4. Evaporation

3 Heat loss occurs by four different mechanisms. In conduction, heat loss occurs when the infant is on a cold surface, such as a table. Radiation occurs when heat from the body surface radiates to the surrounding environment. In convection, air moving across the infant's skin transfers heat to the air. Evaporation of moisture from a wet body surface dissipates heat along with the moisture.

The nurse has provided instructions about measures to clean the penis to a mother of a male newborn who is not circumcised. Which statement, if made by the mother, indicates an understanding of how to clean the newborn's penis? 1. "I should retract the foreskin and clean the penis every time I change the diaper." 2. "I need to retract the foreskin and clean the penis every time I give my infant a bath." 3. "I need to avoid pulling back the foreskin to clean the penis because this may cause adhesions." 4. "I should gently retract the foreskin as far as it will go on the penis and then pull the skin back over the penis after cleaning."

3 In male newborn infants, the prepuce is continuous with the epidermis of the glans and is not retractable. If retraction is forced, this may cause adhesions to develop. The mother should be told to allow separation to occur naturally, which usually occurs between 3 years and puberty. Most foreskins are retractable by 3 years of age and should be pushed back gently at this time for cleaning. Options that identify actions that address retraction of the foreskin are therefore incorrect.

The nurse is performing an initial assessment on a newborn infant. When assessing the infant's head, the nurse notes that the ears are low-set. Which nursing action is most appropriate? 1. Document the findings. 2. Arrange for hearing testing. 3. Notify the health care provider. 4. Cover the ears with gauze pads.

3 Low or oddly placed ears are associated with various congenital defects and should be reported immediately. Although the findings should be documented, the most appropriate action would be to notify the health care provider. Options 2 and 4 are inaccurate and inappropriate nursing actions.

The nurse in the delivery room is performing an assessment on a newborn to determine the Apgar score. The nurse notes an Apgar score of 6. On the basis of this score, what should the nurse determine? 1. The newborn requires vigorous resuscitation. 2. The newborn is adjusting well to extrauterine life. 3. The newborn requires some resuscitative interventions. 4. The newborn is having some difficulty adjusting to extrauterine life.

3 One of the earliest indicators of successful adaptation of the newborn to extrauterine life is the Apgar score. Scoring ranges from 0 to 10. A score of 8 to 10 indicates that the newborn is adjusting well to extrauterine life. A score of 5 to 7 often indicates that the newborn requires some resuscitative interventions. Scores of less than 5 indicate that the newborn is having difficulty adjusting to extrauterine life and requires vigorous resuscitation.

The nurse in the newborn nursery is determining admission vital signs for a newborn infant. The nurse documents that the heart rate is within normal range if which heart rate is noted on assessment? 1. 80 beats/min 2. 90 beats/min 3. 130 beats/min 4. 180 beats/min

3 The normal heart rate for a newborn infant ranges from approximately 120 to 160 beats/min. Options 1 and 2 indicate bradycardia. Option 4 indicates tachycardia.

The nurse is assessing a newborn after circumcision and notes that the circumcised area is red with a small amount of bloody drainage. Which nursing action is most appropriate? 1. Apply gentle pressure. 2. Reinforce the dressing. 3. Document the findings. 4. Contact the health care provider (HCP).

3 The penis is normally red during the healing process after circumcision. A yellow exudate may be noted in 24 hours, and this is part of normal healing. The nurse would expect that the area would be red with a small amount of bloody drainage. Only if the bleeding were excessive would the nurse apply gentle pressure with a sterile gauze. If bleeding cannot be controlled, the blood vessel may need to be ligated, and the nurse would notify the health care provider. Because the findings identified in the question are normal, the nurse would document the assessment findings.

The nurse is developing a plan of care for a preterm newborn infant. The nurse develops measures to provide skin care, knowing that the preterm newborn infant's skin appears in what way? 1. Thin and gelatinous, with increased subcutaneous fat 2. Thin and gelatinous, with increased amounts of brown fat 3. Reddened, translucent, and gelatinous, with decreased amounts of subcutaneous fat 4. With fine downy hair on thin epidermal and dermal layers, with increased amount of brown fat

3 The skin of a newborn infant plays a significant role in thermoregulation and as a barrier against infection. The skin of a preterm newborn infant is immature in comparison with that of a term newborn infant. The skin of a preterm newborn is thin and gelatinous, with decreased amounts of subcutaneous fat, brown fat, and glycogen stores. In addition, preterm newborn infants lose heat because of their large body surface area in relation to their weight and because their posture is more relaxed, with less flexion. Therefore preterm newborn infants are less able to generate heat, which places them at risk for increased heat loss and increased fluid requirements

Which medication should the nurse plan to administer to a newborn by the intramuscular (IM) route? 1. Erythromycin 2. Tetracycline 1% 3. Phytonadione (Vitamin K) 4. Measles-mumps-rubella vaccination

3 Vitamin K is administered intramuscularly into the vastus lateralis muscle. Tetracycline 1% and erythromycin are prescribed for prophylaxis against gonorrhea and are administered into the eye. The measles-mumps-rubella vaccination is not given to a newborn.

The nurse is reviewing the record of a newborn infant in the nursery and notes that the health care provider has documented the presence of a cephalohematoma. Based on this documentation, what should the nurse expect to note on assessment of the infant? 1. A suture split greater than 1 cm 2. A hard, rigid, immobile suture line 3. Swelling of the soft tissues of the head and scalp 4. Edema resulting from bleeding below the periosteum of the cranium

4 A cephalohematoma indicates edema resulting from bleeding below the periosteum of the cranium. It does not cross the suture line. It is most likely to be caused by ruptured blood vessels from head trauma during birth. The lesion develops within 24 to 48 hours after birth and may take 2 to 3 weeks to resolve. Option 1 may indicate increased intracranial pressure. Option 2 may be associated with premature closure or craniosynostosis and should be investigated further. Option 3 identifies a caput succedaneum.

The nurse is performing an admission assessment on a newborn infant with the diagnosis of subdural hematoma after a difficult vaginal delivery. Which assessment technique would assist to support the newborn's diagnosis? 1. Monitoring the urine for blood 2. Monitoring the urinary output pattern 3. Testing for contractures of the extremities 4. Stimulating for reflex responses in the extremities

4 A subdural hematoma can cause pressure on a specific area of the cerebral tissue. Especially if the infant is actively bleeding, such pressure can cause changes in the stimuli responses in the extremities on the opposite side of the body. Options 1 and 2 are incorrect. An infant after delivery normally would be incontinent of urine. Blood in the urine would indicate abdominal trauma and would not be a result of the hematoma. Option 3 is incorrect because contractures would not occur this soon after delivery.

An infant is born to a mother with hepatitis B. Which prophylactic measure would be indicated for the infant? 1. Hepatitis B vaccine given within 24 hours after birth 2. Immune globulin (IG) given as soon as possible after delivery 3. Hepatitis B immune globulin (HBIG) given within 14 days after birth 4. Hepatitis B immune globulin (HBIG) and hepatitis B vaccine given within 12 hours after birth

4 Both HBIG and the vaccine are given to infants with perinatal exposure to prevent hepatitis and achieve lifelong prophylaxis; they are administered within 12 hours after birth. IG is given to prevent hepatitis A.

The nurse is preparing a plan of care for a newborn with fetal alcohol syndrome. The nurse should include which priority intervention in the plan of care? 1. Allow the newborn to establish own sleep-rest pattern. 2. Maintain the newborn in a brightly lighted area of the nursery. 3. Encourage frequent handling of the newborn by staff and parents. 4. Monitor the newborn's response to feedings and weight gain pattern.

4 Fetal alcohol syndrome is caused by maternal alcohol use during pregnancy. A primary nursing goal for the newborn diagnosed with fetal alcohol syndrome is to establish nutritional balance after delivery. These newborns may exhibit hyperirritability, vomiting, diarrhea, or an uncoordinated sucking and swallowing ability. A quiet environment with minimal stimuli and handling would help establish appropriate sleep-rest cycles in the newborn as well. Options 1, 2, and 3 are inappropriate interventions.

The nurse in the labor room is performing an initial assessment on a newborn infant. On assessment of the head, the nurse notes that the ears are low set. Which nursing action would be most appropriate? 1. Document the findings. 2. Arrange for hearing testing. 3. Cover the ears with gauze pads. 4. Notify the health care provider (HCP).

4 Low or oddly placed ears are associated with a variety of congenital defects and should be reported immediately. Although the findings would be documented, the most appropriate action would be to notify the HCP. The remaining options are inaccurate and inappropriate nursing actions.

Methylergonovine (Methergine) has been prescribed for a woman who is at risk for postpartum bleeding in the immediate postpartum period. The nurse preparing to administer the medication ensures that which priority item is at the bedside? 1. Peripads 2. Tape measure 3. Reflex hammer 4. Blood pressure cuff

4 Methylergonovine is an oxytocic agent used to prevent or control postpartum hemorrhage by contracting the uterus. It causes constant uterine contractions and may cause the blood pressure to elevate. A priority assessment before administering this medication is obtaining a baseline blood pressure. The client's blood pressure also should be monitored during the administration of the medication. Methylergonovine is administered cautiously in the presence of hypertension, and the health care provider should be notified if hypertension occurs. The items in options 1, 2, or 3 are not priority items.

The nurse in the labor room measures the Apgar score in a newborn infant and notes that the score is 4. Which action by the nurse has highest priority? 1. Initiate an intravenous (IV) line on the newborn infant. 2. Place the newborn infant on a cardiorespiratory monitor. 3. Place the newborn infant in the radiant warmer incubator. 4. Administer oxygen via resuscitation bag to the newborn infant.

4 Newborn infants with an Apgar score of 5 to 7 often require resuscitative interventions. Scores of less than 5 indicate that the newborn infant is having difficulty adjusting to extrauterine life and requires more vigorous resuscitation. The immediate nursing action should be to administer oxygen via resuscitation bag. Although the newborn infant may require a cardiorespiratory monitor and an IV line and may need to be placed in a radiant warmer incubator, the initial action of the nurse should be to provide resuscitative measures.

The nurse is admitting a newborn infant to the nursery and notes that the health care provider has documented that the newborn has an omphalocele. While performing an assessment, where should the nurse document the location of the viscera in this condition? 1. Inside the abdominal cavity and under the skin 2. Inside the abdominal cavity and under the dermis 3. Outside the abdominal cavity and not covered with a sac 4. Outside the abdominal cavity but inside a translucent sac covered with peritoneum and amniotic membrane

4 Omphalocele is an abdominal wall defect. It involves a large herniation of the gut into the umbilical cord. The viscera are outside the abdominal cavity but inside a translucent sac covered with peritoneum and amniotic membrane. Options 1 and 2 describe an umbilical hernia. Option 3 describes a gastroschisis.

The nurse is performing Apgar scoring for a newborn infant immediately after birth. The nurse notes that the heart rate is greater than 100 beats/min, the respiratory effort is good, muscle tone is active, the newborn infant sneezes when suctioned by the bulb syringe, and the skin color is pink. On the basis of these findings, the nurse should document which Apgar score? 1. 3 2. 5 3. 7 4. 10

4 One of the earliest indicators of successful adaptation of the newborn infant is the Apgar score. Scoring ranges from 0 to 10. Five criteria are used to measure the infant's adaptation. Heart rate: absent = 0; less than 100 beats/min = 1; greater than 100 beats/min = 2. Respiratory effort: absent = 0; slow or irregular weak cry = 1; good, crying lustily = 2. Muscle tone: limp or hypotonic = 0; some extremity flexion = 1; active, moving, and well-flexed = 2. Irritability or reflexes (measured by response to bulb suctioning): no response = 0; grimace = 1; cough, sneeze, or vigorous cry = 2. Color: cyanotic or pale = 0; acrocyanotic, cyanosis of extremities = 1; pink = 2.

Which statement reflects a new mother's understanding of the teaching about the prevention of newborn abduction? 1. "I will place my baby's crib close to the door." 2. "Some health care personnel won't have name badges." 3. "It's OK to allow the unlicensed assistive personnel to carry my newborn to the nursery." 4. "I will ask the nurse to attend to my infant if I am napping and my husband is not here."

4 Precautions to prevent infant abduction include placing a newborn's crib away from the door, transporting a newborn only in the crib and never carrying the newborn, expecting health care personnel to wear identification that is easily visible at all times, and asking the nurse to attend to the newborn if the mother is napping and no family member is available to watch the newborn (the newborn is never left unattended). If the mother states that she will ask the nurse to watch the newborn while she is sleeping, she has understood the teaching. Options 1, 2, and 3 are incorrect and would indicate that the mother needs further teaching.

The home care nurse is visiting a mother 1 week after she gave birth to an infant who is at risk for developing neonatal congenital syphilis. After teaching the mother about the signs and symptoms of this disorder, the nurse instructs the mother to monitor the infant for which finding? 1. Loose stools 2. High-pitched cry 3. Vigorous feeding habits 4. A copper-colored skin rash

4 Signs and symptoms of congenital neonatal syphilis may be nonspecific initially, including poor feedings, slight hyperthermia, and "snuffles." By the end of the first week of life, a copper-colored maculopapular dermal rash typically is observed on the palms of the hands, on the soles of the feet, in the diaper area, and around the mouth and anus. Options 1, 2, and 3 are not associated signs of this disorder.

A nurse employed in a neonatal intensive care nursery receives a telephone call from the delivery room and is told that a newborn with spina bifida (myelomeningocele type) will be transported to the nursery. The maternity nurse prepares for the arrival of the newborn and places which priority item at the newborn's bedside? 1. A rectal thermometer 2. A blood pressure cuff 3. A specific gravity urinometer 4. A bottle of sterile normal saline

4 Spina bifida is a central nervous system defect that results from failure of the neural tube to close during embryonic development. The newborn with spina bifida is at risk for infection before the closure of the sac, which is done soon after birth. A sterile normal saline dressing is placed over the sac to maintain moisture of the sac and its contents. This prevents tearing or breakdown of the skin integrity at the site. A thermometer will be needed to assess temperature, but in this newborn the priority is to maintain sterile normal saline dressings over the sac. Blood pressure may be difficult to assess during the newborn period and is not the best indicator of infection. Urine concentration is not well developed in the newborn stage of development.

A nurse has a routine prescription to instill erythromycin ointment into the eyes of a newborn. The nurse plans to explain to the parents that which is the purpose of the medication? 1. Help the newborn to see more clearly. 2. Ensure the sterility of the conjunctiva in the newborn. 3. Guard against infection acquired during intrauterine life. 4. Protect the newborn from contracting an eye infection during birth.

4 The use of eye prophylaxis with an agent such as erythromycin protects the newborn from contracting a conjunctival infection during birth. This infection, called ophthalmia neonatorum, results from maternal vaginal infection with chlamydia or gonorrhea. This prophylaxis is mandatory in the United States. Options 1, 2, and 3 do not describe the purposes of this medication.

The nurse prepares to administer a vitamin K injection to a newborn, and the mother asks the nurse why her infant needs the injection. What best response should the nurse provide? 1. "Your newborn needs vitamin K to develop immunity." 2. "The vitamin K will protect your newborn from being jaundiced." 3. "Newborns have sterile bowels, and vitamin K promotes the growth of bacteria in the bowel." 4. "Newborns are deficient in vitamin K, and this injection prevents your newborn from bleeding."

4 Vitamin K is necessary for the body to synthesize coagulation factors. Vitamin K is administered to the newborn to prevent bleeding disorders. Vitamin K promotes liver formation of the clotting factors II, VII, IX, and X. Newborns are vitamin K-deficient because the bowel does not have the bacteria necessary for synthesizing fat-soluble vitamin K. The normal flora in the intestinal tract produces vitamin K. The newborn's bowel does not support the normal production of vitamin K until bacteria adequately colonize it. The bowel becomes colonized by bacteria as food is ingested. Vitamin K does not promote the development of immunity or prevent the infant from becoming jaundiced.

The staff nurse in a neonatal intensive care unit is aware that red electrical outlets denote emergency power and will function in the event of an outage. There are only two red outlets in the room of a 4-day-old male newborn being treated for physiological jaundice and to rule out sepsis from group B streptococcal exposure. Which pieces of equipment requiring power would the nurse select to be plugged into the red outlets in case of a power outage? Select all that apply. 1. Call bell 2. Feeding pump 3. Vital sign machine 4. Phototherapy lights 5. Intravenous (IV) pump

4, 5 Given the fact that the newborn is 4 days old, accurate delivery and prevention of circulatory overload is a priority. The IV fluid rate must be maintained using an IV pump. Fluids by gravity would not be the safest mode of delivery in a newborn. The phototherapy lights must be used continually to be effective. The newborn can be fed via gravity using the gavage method if necessary. Vital signs may be obtained without powered equipment. The caregiver may require a call bell, but there are other options for a call device, such as a hand-held noisemaker or whistle.

The nurse is preparing to care for a newborn receiving phototherapy. Which interventions should be included in the plan of care? Select all that apply. 1. Avoid stimulation. 2. Decrease fluid intake. 3. Expose all of the newborn's skin. 4. Monitor skin temperature closely. 5. Reposition the newborn every 2 hours. 6. Cover the newborn's eyes with eye shields or patches.

4, 5, 6 Phototherapy is the use of intense fluorescent lights to reduce serum bilirubin levels in the newborn. Adverse effects from treatment, such as eye damage, dehydration, or sensory deprivation, can occur. Interventions include exposing as much of the newborn's skin as possible; however, the genital area is covered. The newborn's eyes are also covered with eye shields or patches, ensuring that the eyelids are closed when shields or patches are applied. The shields or patches are removed at least once per shift to inspect the eyes for infection or irritation and to allow eye contact. The nurse measures the lamp energy output to ensure efficacy of the treatment (done with a special device known as a photometer), monitors skin temperature closely, and increases fluids to compensate for water loss. The newborn will have loose green stools and green-colored urine. The newborn's skin color is monitored with the fluorescent light turned off every 4 to 8 hours and is monitored for bronze baby syndrome, a grayish brown discoloration of the skin. The newborn is repositioned every 2 hours, and stimulation is provided. After treatment, the newborn is monitored for signs of hyperbilirubinemia because rebound elevations can occur after therapy is discontinued.

The nurse is monitoring a postpartum client who says she's concerned because she feels mildly depressed. The nurse recognizes that she's most likely experiencing "postpartum blues," and reassures the client that this symptom is experienced by approximately what percentage of women? a) 25% b) 100% c) 75% d) 40%

75% Explanation: Postpartum blues, or mild depression during the first 10 days after giving birth, affects 75% to 80% of women who give birth. More intense depression during this period is referred to as postpartum depression, which affects approximately 10% to 15% of postpartum clients. Postpartum depression can be severe with negative implications for maternal and neonatal well-being.

Nurses can help parents deal with the issue and fact of circumcision if they explain a. The pros and cons of the procedure during the prenatal period b. That the American Academy of Pediatrics (AAP) recommends that all newborn males be routinely circumcised c. That circumcision is rarely painful and that any discomfort can be managed without medication d. That the infant will likely be alert and hungry shortly after the procedure

A

Nursing follow-up care often includes home visits for the new mother and her infant. Which information related to home visits is correct? a. Ideally the visit is scheduled between 24 and 72 hours after discharge. b. Home visits are available in all areas. c. Visits are completed within a 30-minute time frame. d. Blood draws are not a part of the home visit.

A

Discharge instruction, or teaching the woman what she needs to know to care for herself and her newborn, officially begins: A. At the time of admission to the nurse's unit B. When the infant is presented to the mother at birth C. During the first visit with the physician in the unit D. When the take-home information packet is given to the couple

A A. Correct: Discharge planning, the teaching of maternal and newborn care, begins on the woman's admission to the unit, continues throughout her stay, and actually never ends as long as she has contact with medical personnel. B. Incorrect: Discharge planning, the teaching of maternal and newborn care, begins on the woman's admission to the unit, continues throughout her stay, and actually never ends as long as she has contact with medical personnel. C. Incorrect: Discharge planning, the teaching of maternal and newborn care, begins on the woman's admission to the unit, continues throughout her stay, and actually never ends as long as she has contact with medical personnel. D. Incorrect: Discharge planning, the teaching of maternal and newborn care, begins on the woman's admission to the unit, continues throughout her stay, and actually never ends as long as she has contact with medical personnel. p. 607

In the recovery room, if a woman is asked either to raise her legs (knees extended) off the bed or to flex her knees, place her feet flat on the bed, and raise her buttocks well off the bed, most likely she is being tested to see whether she: A. Has recovered from epidural or spinal anesthesia B. Has hidden bleeding underneath her C. Has regained some flexibility D. Is a candidate to go home after 6 hours

A A. Correct: If the numb or prickly sensations are gone from her legs after these movements, she likely has recovered from the epidural or spinal anesthesia. B. Incorrect: If the numb or prickly sensations are gone from her legs after these movements, she likely has recovered from the epidural or spinal anesthesia. C. Incorrect: If the numb or prickly sensations are gone from her legs after these movements, she likely has recovered from the epidural or spinal anesthesia. D. Incorrect: If the numb or prickly sensations are gone from her legs after these movements, she likely has recovered from the epidural or spinal anesthesia. p. 587

A primiparous woman is to be discharged from the hospital tomorrow with her infant girl. Which behavior indicates a need for further intervention by the nurse before the woman can be discharged? A. The woman leaves the infant on her bed while she takes a shower. B. The woman continues to hold and cuddle her infant after she has fed her. C. The woman reads a magazine while her infant sleeps. D. The woman changes her infant's diaper and then shows the nurse the contents of the diaper.

A A. Correct: Leaving an infant on a bed unattended is never acceptable for various safety reasons. B. Incorrect: This is an appropriate parent-infant interaction. C. Incorrect: This is an appropriate maternal action. D. Incorrect: This action is appropriate, because the mother is seeking approval from the nurse and notifying the nurse of the infant's elimination patterns. p. 607

A hospital has a number of different perineal pads available for use. A nurse is observed soaking several of them and writing down what she sees. This activity indicates that the nurse is trying to: A. Improve the accuracy of blood loss estimation, which usually is a subjective assessment B. Determine which pad is best C. Demonstrate that other nurses usually underestimate blood loss D. Reveal to the nurse supervisor that one of them needs some time off

A A. Correct: Saturation of perineal pads is a critical indicator of excessive blood loss, and anything done to aid in assessment is valuable. The nurse is noting the saturation volumes and soaking appearances. B. Incorrect: Possibly. More likely the nurse is noting saturation volumes and soaking appearances to improve the accuracy of blood loss estimation. C. Incorrect: Doubtful. Nurses usually overestimate blood loss, if anything. More likely the nurse is noting saturation volumes and soaking appearances to improve the accuracy of blood loss estimation. D. Incorrect: More likely the nurse is noting saturation volumes and soaking appearances to improve the accuracy of blood loss estimation. pp. 597-598

What would prevent early discharge of a postpartum woman? A. Hgb <10 g B. Birth at 38 weeks of gestation C. Voids about 200 to 300 ml per void D. Episiotomy that shows slight redness and edema and is dry and approximated

A A. Correct: The mother's hemoglobin should be above 10 g for early discharge. B. Incorrect: The birth of an infant at term is not a criterion that would prevent early discharge. C. Incorrect: This is a normal voiding volume and does not indicate that the woman should not be discharged early. D. Incorrect: These are normal episiotomy findings and would not prevent a woman from being discharged early. p. 590

While admitting the pregnant woman, the nurse should be aware that postpartum hospital stays that are becoming shorter are primarily the result of the influence of: A. Health maintenance organizations (HMOs) and private insurers B. Consumer demand C. Hospitals D. The federal government

A A. Correct: The trend for shortened hospital stays is based largely on efforts to reduce health care costs. B. Incorrect: Secondarily, consumers have demanded less medical intervention and more family-centered experiences. C. Incorrect: Hospitals are obligated to follow standards of care and federal statutes regarding discharge policies. D. Incorrect: The Newborns' and Mothers' Health Protection Act provided minimum federal standards for health plan coverage for mothers and their newborns. Under this act, couples were allowed to stay in the hospital for longer periods. p. 589

The laboratory results for a postpartum woman are as follows: blood type, A; Rh status, positive; rubella titer, 1:8 (EIA 0.8); hematocrit, 30%. How would the nurse best interpret these data? A. Rubella vaccine should be given. B. A blood transfusion is necessary. C. Rh immune globulin is necessary within 72 hours of birth. D. A Kleihauer-Betke test should be performed.

A A. Correct: This client's rubella titer indicates that she is not immune and that she needs to receive a vaccine. B. Incorrect: These data do not indicate that the client needs a blood transfusion. C. Incorrect: Rh immune globulin is indicated only if the client has a negative Rh status and the infant has a positive Rh status. D. Incorrect: A Kleihauer-Betke test should be performed if a large fetomaternal transfusion is suspected, especially if the mother is Rh negative. The data do not provide any indication for performing this test. p. 603

As recently as 2005, the American Academy of Pediatrics revised safe sleep practices to assist in the prevention of sudden infant death syndrome. The nurse should model these practices in hospital and incorporate this information into the teaching for new parents. They include (select all that apply) a. Fully supine position for all sleep b. Side-sleeping position as an acceptable alternative c. "Tummy time" for play d. Placing the infant's crib in the parents' room e. A soft mattress

A, C, D

Hearing loss occurs in 9% of newborns. Auditory screening of all newborns within the first month of life is recommended by the American Academy of Pediatrics. Reasons for having this testing performed include (select all that apply) a. To prevent or reduce developmental delay b. Reassurance for concerned new parents c. Early identification and treatment d. To help the child communicate better e. To achieve one of the Healthy People 2020 goals

A, C, D, E

Many new mothers experience some type of nipple pain during the first weeks of initiating breastfeeding. Should this pain be severe or persistent, it may discourage or inhibit breastfeeding altogether. The nurse should be aware of a variety of factors that may contribute to nipple pain. These include (choose all that apply): A. Improper feeding position B. Large for gestational age infant C. Fair skin D. Progesterone deficiency E. Flat or retracted nipples

A, C, E Correct Nipple lesions may manifest as chapped, cracked, bleeding, sore, erythematous, edematous, or blistered. Factors that contribute to nipple pain include improper positioning or failure to break suction before removing the baby from the breast. Flat or retracted nipples along with the use of nipple shields, breast shells, or plastic breast pads also contribute. Women with fair skin are more likely to develop sore and cracked nipples. Prevention of nipple soreness is preferable to treatment after it appears. Incorrect Vigorous feeding may be a contributing factor. This may be the case with any size infant, not just those who are large for gestational age. Estrogen or dietary deficiencies can contribute to nipple soreness. p. 600

2. The nurse is performing a blood glucose test every 4 hours on an infant born to a diabetic mother. This is to assess the infant's risk of hypoglycemia. The nurse becomes concerned if the infant's blood glucose concentration falls below ______ mg/dl.

ANS: 40 If the newborn has a blood glucose level below 40 mg/dl intervention such as breastfeeding or bottle-feeding should be instituted. If levels remain low after this intervention an intravenous with dextrose may be warranted.

1. The acronym ________ is used as a reminder that the site of an episiotomy or perineal laceration should be assessed for five physical signs.

ANS: REEDA The acronym REEDA indicates redness, edema, ecchymosis or bruising, discharge, and approximation (the edges of the wound should be close). If redness is accompanied by pain or tenderness, this may indicate infection. Edema may illustrate soft tissue damage and delay wound healing. There should be no discharge. The edges of the wound should be closely approximated as if held together by glue.

3. A ________ succedaneum may appear over the vertex of the newborn's head as a result of pressure against the mother's cervix while in utero.

ANS: caput This pressure causes localized edema and appears as an edematous area on the infant's head. The edema may cross suture lines, is soft to the touch, and varies in size. It usually resolves quickly and disappears entirely within the first few days after birth. Caput may also occur as the result of an operative delivery when a vacuum extractor is used during a vaginal birth.

2. The process in which the uterus returns to a non-pregnant state after birth is known as __________.

ANS: involution This process begins immediately after expulsion of the placenta with contraction of the uterine smooth muscle.

1. The shivering mechanism of heat production is rarely functioning in the newborn. Nonshivering _____________ is accomplished primarily by metabolism of brown fat, which is unique to the newborn, and by increased metabolic activity in the brain, heart, and liver.

ANS: thermogenesis Brown fat is located in superficial deposits in the interscapular region and axillae, as well as in deep deposits at the thoracic inlet, along the vertebral column and around the kidneys. Brown fat has a richer vascular and nerve supply than ordinary fat. Heat produced by intense lipid metabolic activity in brown fat can warm the newborn by increasing heat production by as much as 100%.

32. What is the quickest and most common method to obtain neonatal blood for glucose screening 1 hour after birth? a. Puncture the lateral pad of the heel. b. Obtain a sample from the umbilical cord. c. Puncture a fingertip. d. Obtain a laboratory chemical determination.

ANS: A Feedback A A drop of blood obtained by heel stick is the quickest method of glucose screening. The calcaneus bone should be avoided as osteomyelitis may result from injury to the foot. B Most umbilical cords are clamped in the delivery room and are not available for routine testing. C A neonate's fingertips are too fragile to use for this purpose. D Laboratory chemical determination is the most accurate but the lengthiest method.

23. During which phase of maternal adjustment will the mother relinquish the baby of her fantasies and accept the real baby? a. Letting go b. Taking hold c. Taking in d. Taking on

ANS: A Feedback A Accepting the real infant and relinquishing the fantasy infant occurs during the letting-go phase of maternal adjustment. B During the taking-hold phase the mother assumes responsibility for her own care and shifts her attention to the infant. C In the taking-in phase the mother is primarily focused on her own needs. D There is no taking-on phase of maternal adjustment.

33. A new mother states that her infant must be cold because the baby's hands and feet are blue. The nurse explains that this is a common and temporary condition called a. Acrocyanosis b. Erythema neonatorum c. Harlequin color d. Vernix caseosa

ANS: A Feedback A Acrocyanosis, or the appearance of slightly cyanotic hands and feet, is caused by vasomotor instability, capillary stasis, and a high hemoglobin level. Acrocyanosis is normal and appears intermittently over the first 7 to 10 days. B Erythema toxicum (also called erythema neonatorum) is a transient newborn rash that resembles flea bites. C The harlequin sign is a benign, transient color change in newborns. Half of the body is pale, and the other half is ruddy or bluish red with a line of demarcation. D Vernix caseosa is a cheeselike, whitish substance that serves as a protective covering.

27. In providing support to a new mother who must return to full-time employment 6 weeks after a vaginal delivery, the nurse should a. Allow her to express her positive and negative feelings freely. b. Reassure her that she'll get used to leaving her baby. c. Discuss child care arrangements with her. d. Allow her to solve the problem on her own.

ANS: A Feedback A Allowing the patient to express feelings will provide positive support in her process of maternal adjustment. B This blocks communication and belittles the patient's feelings. C This is an important step in anticipatory guidance, but is not the best way to offer support. D She should be instrumental in solving the problem; however, allowing her time to express her feelings and talk the problem over will assist her in making this decision.

11. Rho immune globulin will be ordered postpartum if which situation occurs? a. Mother Rh-, baby Rh+ b. Mother Rh-, baby Rh- c. Mother Rh+, baby Rh+ d. Mother Rh+, baby Rh-

ANS: A Feedback A An Rh- mother delivering an Rh+ baby may develop antibodies to fetal cells that entered her bloodstream when the placenta separated. The Rho immune globulin works to destroy the fetal cells in the maternal circulation before sensitization occurs. B The blood types are alike, so no antibody formation would be anticipated. C The blood types are alike, so no antibody formation would be anticipated. D If the Rh+ blood of the mother comes in contact with the Rh- blood of the infant, no antibodies would develop because the antigens are in the mother's blood, not the infant's.

9. The nurse should alert the physician when a. The infant is dusky and turns cyanotic when crying. b. Acrocyanosis is present at age 1 hour. c. The infant's blood glucose is 45 mg/dL. d. The infant goes into a deep sleep at age 1 hour.

ANS: A Feedback A An infant who is dusky and becomes cyanotic when crying is showing poor adaptation to extrauterine life. B Acrocyanosis is an expected finding during the early neonatal life. C This is within normal range for a newborn. D Infants enter the period of deep sleep when they are about 1 hour old.

4. Nurses can prevent evaporative heat loss in the newborn by a. Drying the baby after birth and wrapping the baby in a dry blanket b. Keeping the baby out of drafts and away from air conditioners c. Placing the baby away from the outside wall and the windows d. Warming the stethoscope and nurse's hands before touching the baby

ANS: A Feedback A Because the infant is a wet with amniotic fluid and blood, heat loss by evaporation occurs quickly. B Heat loss by convection occurs when drafts come from open doors and air currents created by people moving around. C If the heat loss is caused by placing the baby near cold surfaces or equipment, it is termed a radiation heat loss. D Conduction heat loss occurs when the baby comes in contact with cold objects or surfaces.

17. The mother-baby nurse is able to recognize reciprocal attachment behavior. This refers to a. The positive feedback an infant exhibits toward parents during the attachment process b. Behavior during the sensitive period when the infant is in the quiet alert stage c. Unidirectional behavior exhibited by the infant, initiated and enhanced by eye contact d. Behavior by the infant during the sensitive period to elicit feelings of "falling in love" from the parents

ANS: A Feedback A In this definition, reciprocal refers to the feedback from the infant during the attachment process. B This is a good time for bonding, but it does not define reciprocal attachment. C Reciprocal attachment is not unidirectional. D Reciprocal attachment deals with feedback behavior and is not unidirectional.

28. Which nursing action is designed to avoid unnecessary heat loss in the newborn? a. Place a blanket over the scale before weighing the infant. b. Maintain room temperature at 70° F. c. Undress the infant completely for assessments so they can be finished quickly. d. Take the rectal temperature every hour to detect early changes.

ANS: A Feedback A Padding the scale prevents heat loss from the infant to a cold surface by conduction. B Room temperature should be appropriate to prevent heat loss from convection. Also, if the room is warm enough, radiation will assist in maintaining body heat. C Undressing the infant completely will expose the child to cooler room temperatures and cause a drop in body temperature due to convection. D Hourly assessments are not necessary for a normal newborn with a stable temperature.

7. In fetal circulation, the pressure is greatest in the a. Right atrium b. Left atrium c. Hepatic system d. Pulmonary veins

ANS: A Feedback A Pressure in fetal circulation is greatest in the right atrium, which allows a right-to-left shunting that aids in bypassing the lungs during intrauterine life. B The pressure increases in the left atrium after birth and will close the foramen ovale. C The liver does not filter the blood during fetal life until the end. It is functioning by birth. D Blood bypasses the pulmonary vein during fetal life.

22. Which newborn reflex is elicited by stroking the lateral sole of the infant's foot from the heel to the ball of the foot? a. Babinski b. Tonic neck c. Stepping d. Plantar grasp

ANS: A Feedback A The Babinski reflex causes the toes to flare outward and the big toe to dorsiflex. B The tonic neck reflex (also called the fencing reflex) refers to the posture assumed by newborns when in a supine position. C The stepping reflex occurs when infants are held upright with their heel touching a solid surface and the infant appears to be walking. D Plantar grasp reflex is similar to the palmar grasp reflex: when the area below the toes are touched, the infant's toes curl over the nurse's finger.

2. Which woman is most likely to have severe afterbirth pains and request a narcotic analgesic? a. Gravida 5, para 5 b. Woman who is bottle-feeding her first child c. Primipara who delivered a 7-lb boy d. Woman who wishes to breastfeed as soon as her baby is out of the neonatal intensive care unit

ANS: A Feedback A The discomfort of afterpains is more acute for multiparas because repeated stretching of muscle fibers leads to loss of uterine muscle tone. B Afterpains are particularly severe during breastfeeding, not bottle-feeding. C The uterus of a primipara tends to remain contracted. D The nonnursing mother may have engorgement problems. She should empty her breasts regularly to stimulate milk production so she will have the milk when the baby is strong enough to nurse.

4. Which finding 12 hours after birth requires further assessment? a. The fundus is palpable two fingerbreadths above the umbilicus. b. The fundus is palpable at the level of the umbilicus. c. The fundus is palpable one fingerbreadth below the umbilicus. d. The fundus is palpable two fingerbreadths below the umbilicus.

ANS: A Feedback A The fundus rises to the umbilicus after delivery and remains there for about 24 hours. A fundus that is above the umbilicus may indicate uterine atony or urinary retention. B This is an appropriate assessment finding for 12 hours postpartum. C This is an appropriate assessment finding for 12 hours postpartum. D This is an unusual finding for 12 hours postpartum, but still appropriate.

35. A first-time father is changing the diaper of his 1-day-old daughter. He asks the nurse, "What is this black, sticky stuff in her diaper?" The nurse's best response is a. "That's meconium, which is your baby's first stool. It's normal." b. "That's transitional stool." c. "That means your baby is bleeding internally." d. "Oh, don't worry about that. It's okay."

ANS: A Feedback A This is an accurate statement and the most appropriate response. B Transitional stool is greenish brown to yellowish brown and usually appears by the third day after initiation of feeding. C This statement is not accurate. D This statement is not appropriate. It is belittling to the father and does not educate him about the normal stool patterns of his daughter.

38. The cheeselike, whitish substance that fuses with the epidermis and serves as a protective coating is called a. Vernix caseosa b. Surfactant c. Caput succedaneum d. Acrocyanosis

ANS: A Feedback A This protection is needed because the infant's skin is so thin. B Surfactant is a protein that lines the alveoli of the infant's lungs. C Caput succedaneum is the swelling of the tissue over the presenting part of the fetal head. D Acrocyanosis is cyanosis of the hands and feet, resulting in a blue coloring.

2. Many women given up smoking during pregnancy to protect the health of the fetus. The majority of women resumed smoking within the first 6 months postpartum. Factors that increase the likelihood of relapse include (select all that apply) a. Living with a smoker b. Returning to work c. Weight concerns d. Successful breastfeeding e. Failure to breastfeed

ANS: A, C, E Feedback Correct Other factors include intending to quit for pregnancy only, depression, and stress. Incorrect Successful breastfeeding is likely to inhibit smoking. Returning to work, although stressful, does not necessarily increase a return to smoking.

23. Infants in whom cephalhematomas develop are at increased risk for a. Infection b. Jaundice c. Caput succedaneum d. Erythema toxicum

ANS: B Feedback A Cephalhematomas do not increase the risk for infections. B Cephalhematomas are characterized by bleeding between the bone and its covering, the periosteum. Because of the breakdown of the red blood cells within a hematoma, the infants are at greater risk for jaundice. C Caput is an edematous area on the head from pressure against the cervix. D Erythema toxicum is a benign rash of unknown cause that consists of blotchy red areas.

36. By knowing about variations in infants' blood count, nurses can explain to their patients that a. A somewhat lower than expected red blood cell count could be the result of delay in clamping the umbilical cord. b. The early high white blood cell count (WBC) is normal at birth and should decrease rapidly. c. Platelet counts are higher than in adults for a few months. d. Even a modest vitamin K deficiency means a problem with the blood's ability to clot properly.

ANS: B Feedback A Delayed clamping of the cord results in an increase in hemoglobin and the red blood cell count. B The WBC is high the first day of birth and then declines rapidly. C The platelet count essentially is the same for newborns and adults. D Clotting is sufficient to prevent hemorrhage unless the vitamin K deficiency is significant.

34. The parents of a newborn ask the nurse how much the newborn can see. The parents specifically want to know what type of visual stimuli they should provide for their newborn. The nurse responds to the parents by telling them a. "Infants can see very little until about 3 months of age." b. "Infants can track their parent's eyes and can distinguish patterns; they prefer complex patterns." c. "The infant's eyes must be protected. Infants enjoy looking at brightly colored stripes." d. "It's important to shield the newborn's eyes. Overhead lights help them see better."

ANS: B Feedback A Development of the visual system continues for the first 6 months of life. Visual acuity is difficult to determine, but the clearest visual distance for the newborn appears to be 19 cm. B This is an accurate statement. C Infants prefer to look at complex patterns, regardless of the color. D Infants prefer low illumination and withdraw from bright light.

25. Plantar creases should be evaluated within a few hours of birth because a. The newborn has to be footprinted. b. As the skin dries, the creases will become more prominent. c. Heel sticks may be required. d. Creases will be less prominent after 24 hours.

ANS: B Feedback A Footprinting will not interfere with the creases. B As the infant's skin begins to dry, the creases will appear more prominent, and the infant's gestation could be misinterpreted. C Heel sticks will not interfere with the creases. D The creases will appear more prominent after 24 hours.

16. To assess fundal contraction 6 hours after cesarean delivery, the nurse should a. Palpate forcefully through the abdominal dressing. b. Gently palpate, applying the same technique used for vaginal deliveries. c. Place hands on both sides of the abdomen and press downward. d. Rely on assessment of lochial flow rather than palpating the fundus.

ANS: B Feedback A Forceful palpation should never be used. B Assessment of the fundus is the same for both vaginal and cesarean deliveries; however, palpation should be gentle due to increased discomfort caused by the uterine incision. C The top of the fundus, not the sides, should be palpated and massaged. D The fundus should be palpated and massaged to prevent bleeding.

2. When teaching parents about their newborn's transition to extrauterine life, the nurse explains which organs are nonfunctional during fetal life. They are the a. Kidneys and adrenals b. Lungs and liver c. Eyes and ears d. Gastrointestinal system

ANS: B Feedback A Kidneys and adrenals function during fetal life. The fetus continuously swallows amniotic fluid, which is filtered through the kidneys. B Most of the fetal blood flow bypasses the nonfunctional lungs and liver. C Near term, the eyes are open and the fetus can hear. D The gastrointestinal system functions during fetal life.

9. If the fundus is palpated on the right side of the abdomen above the expected level, the nurse should suspect that the patient has a. Been lying on her right side too long b. A distended bladder c. Stretched ligaments that are unable to support the uterus d. A normal involution

ANS: B Feedback A Position of the patient should not alter uterine position. B The presence of a full bladder will displace the uterus. C The problem is a full bladder displacing the uterus. D This is not an expected finding.

21. The hips of a newborn are examined for developmental dysplasia. Which sign indicates an incomplete development of the acetabulum? a. Negative Ortolani's sign b. Thigh and gluteal creases are asymmetric c. Negative Barlow test d. Knee heights are equal

ANS: B Feedback A Positive Ortolani's sign yields a "clunking" sensation and indicates a dislocated femoral head moving into the acetabulum. B Asymmetric thigh and gluteal creases may indicate potential dislocation of the hip. C During a positive Barlow test, the examiner can feel the femoral head move out of acetabulum. D If the hip is dislocated, the knee on the affected side will be lower.

14. When caring for a newly delivered woman, the nurse is aware that the best measure to prevent abdominal distention after a cesarean birth is a. Rectal suppositories b. Early and frequent ambulation c. Tightening and relaxing abdominal muscles d. Carbonated beverages

ANS: B Feedback A Rectal suppositories can be helpful after distention occurs, but do not prevent it. B Activity can aid the movement of accumulated gas in the gastrointestinal tract. C Ambulation is the best prevention. D Carbonated beverages may increase distention.

19. During which stage of role attainment do the parents become acquainted with their baby and combine parenting activities with cues from the infant? a. Anticipatory b. Formal c. Informal d. Personal

ANS: B Feedback A The anticipatory stage begins during the pregnancy when the parents choose a physician and attend childbirth classes. B A major task of the formal stage of role attainment is getting acquainted with the infant. C The informal stage begins once the parents have learned appropriate responses to their infant's cues. D The personal stage is attained when parents feel a sense of harmony in their role.

26. To promote bonding and attachment immediately after delivery, the nurse should a. Allow the mother quiet time with her infant. b. Assist the mother in assuming an en face position with her newborn. c. Teach the mother about the concepts of bonding and attachment. d. Assist the mother in feeding her baby.

ANS: B Feedback A The mother should be given as much privacy as possible; however, nursing assessments must still be continued during this critical time. B Assisting the mother in assuming an en face position with her newborn will support the bonding process. C The mother has just delivered and is more focused on the infant; she will not be receptive to teaching at this time. D This is a good time to initiate breastfeeding, but first the mother needs time to explore the new infant and begin the bonding process.

8. Cardiovascular changes that cause the foramen ovale to close at birth are a direct result of a. Increased pressure in the right atrium b. Increased pressure in the left atrium c. Decreased blood flow to the left ventricle d. Changes in the hepatic blood flow

ANS: B Feedback A The pressure in the right atrium decreases at birth. It is higher during fetal life. B With the increase in the blood flow to the left atrium from the lungs, the pressure is increased, and the foramen ovale is functionally closed. C Blood flow increases to the left ventricle after birth. D The hepatic blood flow changes, but that is not the reason for the closure of the foramen ovale.

3. A woman gave birth to a healthy 7-pound, 13-ounce infant girl. The nurse suggests that the woman place the infant to her breast within 15 minutes after birth. The nurse knows that breastfeeding is effective during the first 30 minutes after birth because this is the a. Transition period b. First period of reactivity c. Organizational stage d. Second period of reactivity

ANS: B Feedback A The transition period is the phase between intrauterine and extrauterine existence. B The first period of reactivity is the first phase of transition and lasts up to 30 minutes after birth. The infant is highly alert during this phase. C There is no such phase as the organizational stage. D The second period of reactivity occurs roughly between 4 and 8 hours after birth, after a period of prolonged sleep.

25. A man calls the nurse's station stating that his wife, who delivered 2 days ago, is happy one minute and crying the next. The man says, "She was never like this before the baby was born." The nurse's initial response should be to a. Tell him to ignore the mood swings, as they will go away. b. Reassure him that this behavior is normal. c. Advise him to get immediate psychological help for her. d. Instruct him in the signs, symptoms, and duration of postpartum blues.

ANS: B Feedback A This blocks communication and may belittle the husband's concerns. B Before providing further instructions, inform family members of the fact that postpartum blues are a normal process to allay anxieties and increase receptiveness to learning. C Postpartum blues are a normal process that is short lived; no medical intervention is needed. D Client teaching is important; however, his anxieties need to be allayed before he will be receptive to teaching.

27. A new mother asks, "Why are you doing a gestational age assessment on my baby?" The nurse's best response is a. "This must be done to meet insurance requirements." b. "It helps us identify infants who are at risk for any problems." c. "The gestational age determines how long the infant will be hospitalized." d. "It was ordered by your doctor."

ANS: B Feedback A This is not accurate information. B The nurse should provide the mother with accurate information about various procedures performed on the newborn. C Gestational age does not dictate hospital stays. Problems that occur due to gestational age may prolong the stay. D Assessing gestational age is a nursing assessment and does not have to be ordered.

16. The process in which bilirubin is changed from a fat-soluble product to a water-soluble product is known as a. Enterohepatic circuit b. Conjugation of bilirubin c. Unconjugation of bilirubin d. Albumin binding

ANS: B Feedback A This is the route by which part of the bile produced by the liver enters the intestine, is reabsorbed by the liver, and then is recycled into the intestine. B Conjugation of bilirubin is the process of changing the bilirubin from a fat-soluble to a water-soluble product. C Unconjugated bilirubin is fat soluble. D Albumin binding is to attach something to a protein molecule.

18. The postpartum woman who continually repeats the story of her labor, delivery, and recovery experiences is a. Providing others with her knowledge of events b. Making the birth experience "real" c. Taking hold of the events leading to her labor and delivery d. Accepting her response to labor and delivery

ANS: B Feedback A This is to satisfy her needs, not others. B Reliving the birth experience makes the event real and helps the mother realize that the pregnancy is over and that the infant is born and is now a separate individual. C She is in the taking-in phase, trying to make the birth experience seem real. D She is trying to make the event real and is trying to separate the infant from herself.

1. What are modes of heat loss in the newborn? (Choose all that apply.) a. Perspiration b. Convection c. Radiation d. Conduction e. Urination

ANS: B, C, D Feedback Correct Convection, radiation, evaporation, and conduction are the four modes of heat loss in the newborn. Incorrect Perspiration and urination are not modes of heat loss in newborns.

24. A maculopapular rash with a red base and a small white papule in the center is a. Milia b. Mongolian spots c. Erythema toxicum d. Cafe-au-lait spots

ANS: C Feedback A Milia are minute epidermal cysts on the face of the newborn. B Mongolian spots are bluish-black discolorations found on dark-skinned newborns, usually on the sacrum. C This is a description of erythema toxicum, a normal rash in the newborn. D These spots are pale tan (the color of coffee with milk) macules. Occasional spots occur normally in newborns.

1. Nurses must be aware of the conditions that increase the risk of hemorrhage, one of the most common complications of the puerperium. What are the conditions? Select all that apply. a. Primipara b. Rapid or prolonged labor c. Overdistention of the uterus d. Uterine fibroids e. Preeclampsia

ANS: B, C, D, E Feedback Correct Rapid or prolonged labor, overdistention of the uterus, uterine fibroids, and preeclampsia are all risk factors for postpartum hemorrhage. Incorrect Grand multiparity (5 or more pregnancies) is a risk factor for postpartum hemorrhage. Other risk factors include retained placenta, placenta previa, previous postpartum hemorrhage or placenta accreta, drugs (magnesium sulfate, tocolytics, oxytocin), and operative procedures.

33. Two days ago, a woman gave birth to a full-term infant. Last night, she awakened several times to urinate and noted that her gown and bedding were wet from profuse diaphoresis. One mechanism for the diaphoresis and diuresis that this woman is experiencing during the early postpartum period is a. Elevated temperature caused by postpartum infection b. Increased basal metabolic rate after giving birth c. Loss of increased blood volume associated with pregnancy d. Increased venous pressure in the lower extremities

ANS: C Feedback A An elevated temperature causes chills and may cause dehydration, not diaphoresis and diuresis. B Diaphoresis and diuresis sometimes are referred to as reversal of the water metabolism of pregnancy, not as the basal metabolic rate. C Within 12 hours of birth, women begin to lose the excess tissue fluid that has accumulated during pregnancy. One mechanism for reducing these retained fluids is the profuse diaphoresis that often occurs, especially at night, for the first 2 or 3 days after childbirth. Postpartal diuresis is another mechanism by which the body rids itself of excess fluid. D Postpartal diuresis may be caused by the removal of increased venous pressure in the lower extremities.

26. A newborn who is large for gestational age (LGA) is _____ percentile for weight. a. Below the 90th b. Less than the 10th c. Greater than the 90th d. Between the 10th and 90th

ANS: C Feedback A An infant between the 10th and 90th percentiles is average for gestational age. B An infant in less than the 10th percentile is small for gestational age. C The LGA rating is based on weight and is defined as greater than the 90th percentile in weight. D This infant is considered average for gestational age.

1. A postpartum woman overhears the nurse tell the obstetrics clinician that she has a positive Homans sign and asks what it means. The nurse's best response is a. "You have pitting edema in your ankles." b. "You have deep tendon reflexes rated 2+." c. "You have calf pain when the nurse flexes your foot." d. "You have a 'fleshy' odor to your vaginal drainage."

ANS: C Feedback A Edema is within normal limits for the first few days until the excess interstitial fluid is remobilized and excreted. B Deep tendon reflexes should be 1+ to 2+. C Discomfort in the calf with sharp dorsiflexion of the foot may indicate a deep vein thrombosis. D A "fleshy" odor, not a foul odor, is within normal limits.

32. Which hormone remains elevated in the immediate postpartum period of the breastfeeding woman? a. Estrogen b. Progesterone c. Prolactin d. Human placental lactogen

ANS: C Feedback A Estrogen and progesterone levels decrease markedly after expulsion of the placenta, reaching their lowest levels 1 week into the postpartum period. B Estrogen and progesterone levels decrease markedly after expulsion of the placenta, reaching their lowest levels 1 week into the postpartum period. C Prolactin levels in the blood increase progressively throughout pregnancy. In women who breastfeed, prolactin levels remain elevated into the sixth week after birth. D Human placental lactogen levels dramatically decrease after expulsion of the placenta.

8. A pregnant patient asks when the dark line on her abdomen (linea nigra) will go away. The nurse knows the pigmentation will decrease after delivery because of a. Increased estrogen b. Increased progesterone c. Decreased melanocyte-stimulating hormone d. Decreased human placental lactogen

ANS: C Feedback A Estrogen levels decrease after delivery. B Progesterone levels decrease after delivery. C Melanocyte-stimulating hormone increases during pregnancy and is responsible for changes in skin pigmentation; the amount decreases after delivery. D Human placental lactogen production continues to aid in lactation. However, it does not affect pigmentation.

29. What characteristic shows the greatest gestational maturity? a. Few rugae on the scrotum and testes high in the scrotum b. Infant's arms and legs extended c. Some peeling and cracking of the skin d. The arm can be positioned with the elbow beyond the midline of the chest

ANS: C Feedback A Few rugae on the scrotum show a younger age in the newborn. B Extended arms and legs is a sign of preterm infants. C Peeling, cracking, dryness, and a few visible veins in the skin are signs of maturity in the newborn. D This result of the scarf sign shows a younger newborn.

22. The best way for the nurse to promote and support the maternal-infant bonding process is to a. Help the mother identify her positive feelings toward the newborn. b. Encourage the mother to provide all newborn care. c. Assist the family with rooming-in. d. Return the newborn to the nursery during sleep periods.

ANS: C Feedback A Having the mother express her feelings is important, but it is not the best way to promote bonding. B The mother needs time to rest and recuperate; she should not be expected to do all of the care. C Close and frequent interaction between mother and infant, which is facilitated by rooming-in, is important in the bonding process. This is often referred to as the mother-baby care or couplet care. D The mother needs to observe the infant during all stages so she will be aware of what to expect when they go home.

19. In which infant behavioral state is bonding most likely to occur? a. Drowsy b. Active alert c. Quiet alert d. Crying

ANS: C Feedback A In the drowsy state the eyes may remain closed. If open they are unfocused. The infant is not interested in the environment at this time. B In the active alert state infants are often fussy, restless, and not focused. C In the quiet alert state, the infant is interested in his or her surroundings and will often gaze at the mother or father or both. D During the crying state the infant does not respond to stimulation and cannot focus on parents.

14. A meconium stool can be differentiated from a transitional stool in the newborn because the meconium stool is a. Seen at age 3 days b. The residue of a milk curd c. Passed in the first 12 hours of life d. Lighter in color and looser in consistency

ANS: C Feedback A Meconium stool is the first stool of the newborn. B Meconium stool is made up of matter in the intestines during intrauterine life. C Meconium stool is usually passed in the first 12 hours of life and 99% of newborns have their first stool within 48 hours. If meconium is not passed by 48 hours, obstruction is suspected. D Meconium is dark in color and sticky.

5. A first-time dad is concerned that his 3-day-old daughter's skin looks "yellow." In the nurse's explanation of physiologic jaundice, what fact should be included? a. Physiologic jaundice occurs during the first 24 hours of life. b. Physiologic jaundice is caused by blood incompatibilities between the mother and infant blood types. c. The bilirubin levels of physiologic jaundice peak between the second and fourth days of life. d. This condition is also known as "breast milk jaundice."

ANS: C Feedback A Pathologic jaundice occurs during the first 24 hours of life. B Pathologic jaundice is caused by blood incompatibilities, causing excessive destruction of erythrocytes, and must be investigated. C Physiologic jaundice becomes visible when the serum bilirubin reaches a level of 5 mg/dL or greater, which occurs when the baby is approximately 3 days old. This finding is within normal limits for the newborn. D Breast milk jaundice occurs in one third of breastfed infants at 2 weeks and is caused by an insufficient intake of fluids.

11. What is a result of hypothermia in the newborn? a. Shivering to generate heat b. Decreased oxygen demands c. Increased glucose demands d. Decreased metabolic rate

ANS: C Feedback A Shivering is not an effective method of heat production for newborns. B Oxygen demands increase with hypothermia. C In hypothermia, the basal metabolic rate (BMR) is increased in an attempt to compensate, thus requiring more glucose. D The metabolic rate increases with hypothermia.

5. If the patient's white blood cell (WBC) count is 25,000/mm3 on her second postpartum day, the nurse should a. Tell the physician immediately. b. Have the laboratory draw blood for reanalysis. c. Recognize that this is an acceptable range at this point postpartum. d. Begin antibiotic therapy immediately.

ANS: C Feedback A Since this is a normal finding there is no reason to alert the physician. B There is no need for reassessment since it is expected for the WBCs to be elevated. C Marked leucocytosis occurs with WBC counts increasing to as high as 30,000/mm3during labor and the immediate postpartum period. The WBC falls to normal within 6 days postpartum. D Antibiotics are not needed because the elevated WBCs are due to stress of labor and not an infectious process.

6. To provide competent newborn care, the nurse understands that respirations are initiated at birth as a result of a. An increase in the PO2 and a decrease in PCO2 b. The continued functioning of the foramen ovale c. Chemical, thermal, sensory, and mechanical factors d. Drying off the infant

ANS: C Feedback A The PO2 decreases at birth and the PCO2 increases. B The foramen ovale closes at birth. C A variety of these factors are responsible for initiation of respirations. D Tactile stimuli aid in initiating respirations, but are not the main cause.

17. Which statement is correct regarding the fluid balance in a newborn versus that in an adult? a. The infant has a smaller percentage of surface area to body mass. b. The infant has a smaller percentage of water to body mass. c. The infant has a greater percentage of insensible water loss. d. The infant has a 50% more effective glomerular filtration rate.

ANS: C Feedback A The infant's surface area is large compared to an adult's. B Infants have a larger percentage of water to body mass. C Insensible water loss is greater in the infant due to the newborn's large body surface area and rapid respiratory rate. D The filtration rate is less than in adults; the kidneys are immature in a newborn.

15. What documentation on a woman's chart on postpartum day 14 indicates a normal involution process? a. Moderate bright red lochial flow b. Breasts firm and tender c. Fundus below the symphysis and not palpable d. Episiotomy slightly red and puffy

ANS: C Feedback A The lochia should be changed by this day to serosa. B Breasts are not part of the involution process. C The fundus descends 1 cm/day, so by postpartum day 14 it is no longer palpable. D The episiotomy should not be red or puffy at this stage.

10. While assessing the newborn, the nurse should be aware that the average expected apical pulse range of a full-term, quiet, alert newborn is _____ beats/min. a. 80 to 100 b. 100 to 120 c. 120 to 160 d. 150 to 180

ANS: C Feedback A The newborn's heart rate may be about 85 to 100 beats/min while sleeping. B The infant's heart rate typically is a bit higher when alert but quiet. C The average infant heart rate while awake is 120 to 160 beats/min. D A heart rate of 150 to 180 beats/min is typical when the infant cries.

37. With regard to the newborn's developing cardiovascular system, nurses should be aware that a. The heart rate of a crying infant may rise to 120 beats/min. b. Heart murmurs heard after the first few hours are cause for concern. c. The point of maximal impulse (PMI) often is visible on the chest wall. d. Persistent bradycardia may indicate respiratory distress syndrome (RDS).

ANS: C Feedback A The normal heart rate for infants who are not sleeping is 120 to 160 beats/min. However, a crying infant temporarily could have a heart rate of 180 beats/min. B Heart murmurs during the first few days of life have no pathologic significance; an irregular heart rate past the first few hours should be evaluated further. C The newborn's thin chest wall often allows the PMI to be seen. D Persistent tachycardia may indicate RDS; bradycardia may be a sign of congenital heart blockage.

6. Postpartal overdistention of the bladder and urinary retention can lead to which complication? a. Postpartum hemorrhage and eclampsia b. Fever and increased blood pressure c. Postpartum hemorrhage and urinary tract infection d. Urinary tract infection and uterine rupture

ANS: C Feedback A There is no correlation between bladder distention and eclampsia. B There is no correlation between bladder distention and blood pressure or fevers. C Incomplete emptying and overdistention of the bladder can lead to urinary tract infection. Overdistention of the bladder displaces the uterus and prevents contraction of the uterine muscle. D The risk of uterine rupture decreases after the birth.

20. Heat loss by convection occurs when a newborn is a. Placed on a cold circumcision board b. Given a bath c. Placed in a drafty area of the room d. Wrapped in cool blankets

ANS: C Feedback A This is conduction. B This is evaporation. C Convection occurs when infants are exposed to cold air currents. D This is conduction.

18. The most likely interpretation of an elevated immunoglobulin M (IgM) level in a newborn is a. The infant was breastfed during the first hours after birth b. Transference of immune globulins from the placenta to the infant c. An overwhelming allergic response to an antigen d. A recent exposure to a pathogenic agent

ANS: D Feedback A This is the IgA. B This is the IgG. C This is not associated with elevated levels of IgM. D An elevated level of IgM is associated with exposure to infection in utero because IgM does not cross the placenta.

20. On observing a woman on her first postpartum day sitting in bed while her newborn lies awake in the bassinet, the nurse should a. Realize that this situation is perfectly acceptable. b. Offer to hand the baby to the woman. c. Hand the baby to the woman. d. Explain "taking in" to the woman.

ANS: C Feedback A This is expected behavior during the taking-in phase. However, interventions can facilitate infant bonding. B The woman is dependent and passive at this stage and may have difficulty making a decision. C During the "taking-in" phase of maternal adaptation, in which the mother may be passive and dependent, the nurse should encourage bonding when the infant is in the quiet alert stage. This is done best by simply giving the baby to the mother. D She learns best during the taking-hold phase.

13. In administering vitamin K to the infant shortly after birth, the nurse understands that vitamin K is a. Important in the production of red blood cells b. Necessary in the production of platelets c. Not initially synthesized because of a sterile bowel at birth d. Responsible for the breakdown of bilirubin and prevention of jaundice

ANS: C Feedback A Vitamin K is important for blood clotting. B The platelet count in term newborns is near adult levels. Vitamin K is necessary to activate prothrombin and other clotting factors. C The bowel is initially sterile in the newborn, and vitamin K cannot be synthesized until food is introduced into the bowel. D Vitamin K is necessary to activate the clotting factors.

10. A woman gave birth vaginally to a 9-pound, 12-ounce girl yesterday. Her primary health care provider has written orders for perineal ice packs, use of a sitz bath tid, and a stool softener. What information is most closely correlated with these orders? a. The woman is a gravida 2, para 2. b. The woman had a vacuum-assisted birth. c. The woman received epidural anesthesia. d. The woman has an episiotomy.

ANS: D Feedback A A multiparous classification is not an indication for these orders. B A vacuum-assisted birth may be used in conjunction with an episiotomy, which indicates these interventions. C Use of epidural anesthesia has no correlation with these orders. D These orders are typical interventions for a woman who has had an episiotomy, lacerations, and hemorrhoids.

31. The nurse caring for the postpartum woman understands that breast engorgement is caused by a. Overproduction of colostrum b. Accumulation of milk in the lactiferous ducts and glands c. Hyperplasia of mammary tissue d. Congestion of veins and lymphatics

ANS: D Feedback A Breast engorgement is not the result of overproduction of colostrum. B Accumulation of milk in the lactiferous ducts and glands does not cause breast engorgement. C Hyperplasia of mammary tissue does not cause breast engorgement. D Breast engorgement is caused by the temporary congestion of veins and lymphatics.

12. The infant with the lowest risk of developing high levels of bilirubin is the one who a. Was bruised during a difficult delivery b. Developed a cephalhematoma c. Uses brown fat to maintain temperature d. Breastfeeds during the first hour of life

ANS: D Feedback A Bruising will release more bilirubin into the system. B Cephalhematomas will release bilirubin into the system as the red blood cells die off. C Brown fat is normally used to produce heat in the newborn. D The infant who is fed early will be less likely to retain meconium and reabsorb bilirubin from the intestines back into the circulation.

29. A 25-year-old multiparous woman gave birth to an infant boy 1 day ago. Today her husband brings a large container of brown seaweed soup to the hospital. When the nurse enters the room, the husband asks for help with warming the soup so that his wife can eat it. The nurse's most appropriate response is to ask the woman a. "Didn't you like your lunch?" b. "Does your doctor know that you are planning to eat that?" c. "What is that anyway?" d. "I'll warm the soup in the microwave for you."

ANS: D Feedback A Cultural dietary preferences must be respected. B Women may request that family members bring favorite or culturally appropriated foods to the hospital. C Cultural dietary preferences must be respected. A statement such as this does not show cultural sensitivity. D This statement shows cultural sensitivity to the dietary preferences of the woman and is the most appropriate response.

35. Childbirth may result in injuries to the vagina and uterus. Pelvic floor exercises also known as Kegel exercises will help to strengthen the perineal muscles and encourage healing. The nurse knows that the patient understands the correct process for completing these conditioning exercises when she reports a. "I contract my thighs, buttocks, and abdomen." b. "I do 10 of these exercises every day." c. "I stand while practicing this new exercise routine." d. "I pretend that I am trying to stop the flow of urine midstream."

ANS: D Feedback A Each contraction should be as intense as possible without contracting the abdomen, buttocks, or thighs. B Guidelines suggest that these exercises should be done 24 to 100 times per day. Positive results are shown with a minimum of 24 to 45 repetitions per day. C The best position to learn Kegel exercises is to lie supine with knees bent. A secondary position is on the hands and knees. D The woman can pretend that she is attempting to stop the passing of gas, or the flow of urine midstream. This will replicate the sensation of the muscles drawing upward and inward.

28. A new father states, "I know nothing about babies," but he seems to be interested in learning. The nurse should a. Continue to observe his interaction with the newborn. b. Tell him when he does something wrong. c. Show no concern, as he will learn on his own. d. Include him in teaching sessions.

ANS: D Feedback A It is important to note the bonding process of the mother and the father, but that does not satisfy the expressed needs of the father. B He should be encouraged by pointing out the correct procedures he does. Criticizing him will discourage him. C This is not a nursing role. Nurses need to be sensitive to patients' needs. D The nurse must be sensitive to the father's needs and include him whenever possible. As fathers take on care new role, the nurse should praise every attempt even if his early care is awkward.

31. An African-American woman noticed some bruises on her newborn girl's buttocks. She asks the nurse who spanked her daughter. The nurse explains that these marks are called a. Lanugo b. Vascular nevi c. Nevus flammeus d. Mongolian spots

ANS: D Feedback A Lanugo is the fine, downy hair seen on a term newborn. B A vascular nevus, commonly called a strawberry mark, is a type of capillary hemangioma. C A nevus flammeus, commonly called a port-wine stain, is most frequently found on the face. D A Mongolian spot is a bluish black area of pigmentation that may appear over any part of the exterior surface of the body. It is more commonly noted on the back and buttocks and most frequently is seen on infants whose ethnic origins are Mediterranean, Latin American, Asian, or African.

13. Which nursing action is most appropriate to correct a boggy uterus that is displaced above and to the right of the umbilicus? a. Notify the physician of an impending hemorrhage. b. Assess the blood pressure and pulse. c. Evaluate the lochia. d. Assist the patient in emptying her bladder.

ANS: D Feedback A Nursing actions need to be implemented before notifying the physician. B This is an important assessment if the bleeding continues. However, the focus should be on controlling the bleeding. C The focus needs to be on controlling the bleeding. D Urinary retention can cause overdistention of the urinary bladder, which lifts and displaces the uterus.

30. Because a full bladder prevents the uterus from contracting normally, nurses intervene to help the woman empty her bladder spontaneously as soon as possible. If all else fails, the last thing the nurse might try is a. Pouring water from a squeeze bottle over the woman's perineum b. Providing hot tea c. Asking the physician to prescribe analgesics d. Inserting a sterile catheter

ANS: D Feedback A Pouring water over the perineum may stimulate voiding. It is easy, noninvasive, and should be tried early on. B Hot tea or other fluids ad lib is an easy, noninvasive strategy, that should be tried early on. C If the woman is anticipating pain from voiding, pain medications may be helpful. Other nonmedical means could be tried first, but medications still come before insertion of a catheter. D Invasive procedures usually are the last to be tried, especially with so many other simple and easy methods available (e.g., water, peppermint vapors, pain pills).

7. A postpartum patient asks, "Will these stretch marks go away?" The nurse's best response is a. "They will continue to fade and should be gone by your 6-week checkup." b. "No, never." c. "Yes, eventually." d. "They will fade to silvery lines but won't disappear completely."

ANS: D Feedback A Stretch marks do not disappear. B This is true, but more information can be added, such as the changes that will occur with the stretch marks. C This is not a true statement; they will not disappear. D Stretch marks never disappear altogether, but they gradually fade to silvery lines.

3. Which maternal event is abnormal in the early postpartum period? a. Diuresis and diaphoresis b. Flatulence and constipation c. Extreme hunger and thirst d. Lochial color changes from rubra to alba

ANS: D Feedback A The body rids itself of increased plasma volume. Urine output of 3000 mL/day is common for the first few days after delivery and is facilitated by hormonal changes in the mother. B Bowel tone remains sluggish for days. Many women anticipate pain during defecation and are unwilling to exert pressure on the perineum. C The new mother is hungry because of energy used in labor and thirsty because of fluid restrictions during labor. D For the first 3 days after childbirth, lochia is termed rubra. Lochia serosa follows, and then at about 11 days, the discharge becomes clear, colorless, or white.

15. When the newborn infant is fed, the most likely cause of regurgitation is a. Placing the infant in a prone position after a feeding b. The gastrocolic reflex c. An underdeveloped pyloric sphincter d. A relaxed cardiac sphincter

ANS: D Feedback A The infant should be placed in a supine position. B The gastrocolic reflex increases intestinal peristalsis after the stomach fills. C The pyloric sphincter goes from the stomach to the intestines. D The underlying cause of newborn regurgitation is a relaxed cardiac sphincter.

12. If rubella vaccine is indicated for a postpartum patient, instructions to the patient should include a. Drinking plenty of fluids to prevent fever b. No specific instructions c. Recommending that she stop breastfeeding for 24 hours after injection d. Explaining the risks of becoming pregnant within 1 month after injection

ANS: D Feedback A The mother should be afebrile before the vaccine. B The mother does need to understand potential side effects, and that pregnancy is discouraged for at least 28 days after receiving the vaccine. C Small amounts of the vaccine do cross the breast milk, but it is believed that there is no need to discontinue breastfeeding. D Potential risks to the fetus can occur if pregnancy results within 28 days after rubella vaccine administration.

21. A nurse is observing a family. The mother is holding the baby she delivered less than 24 hours ago. Her husband is watching his wife and asking questions about newborn care. The 4-year-old brother is punching his mother on the back. The nurse should a. Report the incident to the social services department. b. Advise the parents that the toddler needs to be reprimanded. c. Report to oncoming staff that the mother is probably not a good disciplinarian. d. Realize that this is a normal family adjusting to family change.

ANS: D Feedback A There is no need to report this one incident. B Giving advice at this point would make the parents feel inadequate as parents. C This is normal for an adjusting family. D The observed behaviors are normal variations of families adjusting to change.

1. A nursing student is helping the nursery nurses with morning vital signs. A baby born 10 hours ago via cesarean section is found to have moist lung sounds. What is the best interpretation of these data? a. The nurse should notify the pediatrician stat for this emergency situation. b. The neonate must have aspirated surfactant. c. If this baby was born vaginally, it could indicate a pneumothorax. d. The lungs of a baby delivered by cesarean section may sound moist for 24 hours after birth.

ANS: D Feedback A This is a common condition for infants delivered by cesarean section. B Surfactant is produced by the lungs, so aspiration is not a concern. C It is common to have some fluid left in the lungs; this will be absorbed within a few hours. D The condition will resolve itself within a few hours. For this common condition of newborns, surfactant acts to keep the expanded alveoli partially open between respirations. In vaginal births, absorption of remaining lung fluid is accelerated by the process of labor and delivery. Remaining lung fluid will move into interstitial spaces and be absorbed by the circulatory and lymphatic systems.

24. A 25-year-old gravida 1 para 1 who had an emergency cesarean birth 3 days ago is scheduled for discharge. As you prepare her for discharge, she begins to cry. Your initial action should be to a. Assess her for pain. b. Point out how lucky she is to have a healthy baby. c. Explain that she is experiencing postpartum blues. d. Allow her time to express her feelings.

ANS: D Feedback A This is an assumption that she is in pain. B This is blocking communication. C She needs the opportunity to express her feelings first. Later, patient teaching can occur. D Although many women experience transient postpartum blues, they need assistance in expressing their feelings. This condition affects 70-80% of new mothers.

30. A sign of illness in the newborn is a. More than two soft stools per day b. Regurgitating a small amount of feeding c. A yellow scaly lesion on the scalp d. An axillary temperature greater than 37.5° C

ANS: D Feedback A This is an expected finding in the newborn. B This is an expected finding in the newborn. C This is a sign of cradle cap or seborrhea capitis. D Infants commonly respond to a variety of illnesses with an elevation in temperature. The normal range for an axillary temperature in the newborn is 36.5° to 37.3° C.

34. Which condition, not uncommon in pregnancy, is likely to require careful medical assessment during the puerperium? a. Varicosities of the legs b. Carpal tunnel syndrome c. Periodic numbness and tingling of the fingers d. Headaches

ANS: D Feedback A Total or nearly total regression of varicosities is expected after childbirth. However, headaches might deserve attention. B Carpal tunnel syndrome is relieved in childbirth when the compression on the median nerve is lessened. Headaches, however, might deserve attention. C Periodic numbness of the fingers usually disappears after birth unless carrying the baby aggravates the condition. Headaches, however, might deserve attention. D Headaches in the postpartum period can have a number of causes, some of which deserve medical attention.

2. Clotting factors and fibrinogen levels normally are decreased during pregnancy and remain low in the immediate puerperium. This hypocoagulable state increases the risk of thromboembolism, especially after cesarean birth. Is this statement true or false?

ANS: F Clotting factors and fibrinogen normally are increased during pregnancy and remain elevated in the immediate puerperium. This hypercoagulable state increases the risk of thromboembolism, especially after cesarean birth.

1. Part of the newborn assessment includes examination of the umbilical cord. The cord should contain 2 vessels: one vein and one artery. Is this statement true or false?

ANS: F The umbilical cord contains 3 vessels: two small arteries and one large vein. A 2-vessel cord may be an isolated abnormality or it may be associated with chromosomal and renal defects.

2. In many facilities protocols allow the nurses to obtain transcutaneous bilirubin measurements (TcB) using a bilirubin meter, without the order of a nurse practitioner or physician. Is this statement true or false?

ANS: T Bilirubinometers are non-invasive devices to measure bilirubin levels in the infant's skin, thus avoiding repeated skin punctures to obtain blood samples. Abnormal results of TcB be should be confirmed with a total serum bilirubin (TsB). The National Association of Neonatal Nurses recommends obtaining a TcB or TsB on all infants prior to discharge.

1. The nurse evaluating the amount of lochia on a newly delivered patient knows that a moderate amount of flow constitutes a 4- to 6-inch stain on the peripad. Is this statement true or false?

ANS: T Since estimating the amount of lochia is difficult, nurses frequently record flow by estimating the amount of lochia in 1 hour using the following labels: Scant—less than a 1-inch stain on the peripad Light—a 1- to 4-inch stain Moderate—a 4- to 6-inch stain Heavy—saturated peripad Excessive—saturated peripad in 15 minutes Determining the time interval that the peripad is in place is also important. Lochia is less for women who have had a cesarean birth since some of the endometrial lining is removed during surgery.

A nurse is caring for a client who gave birth about 10 hours earlier. The nurse observes perineal edema in the client. What intervention should the nurse perform to decrease the swelling caused due to perineal edema? a) Apply moist heat b) Use a warm sitz bath or tub bath c) Apply ice d) Use ointments locally

Apply ice Correct Explanation: Ice is applied to perineal edema within 24 hours after delivery. Use of ointments is not advised for perineal edema. Moist heat and a sitz or tub bath are encouraged if edema continues 24 hours after delivery.

Which intervention would be helpful to a bottle-feeding client who's experiencing hard or engorged breasts? a) Administering bromocriptine (Parlodel) b) Applying warm compresses c) Restricting fluids d) Applying ice

Applying ice Correct Explanation: Ice promotes comfort by decreasing blood flow (vasoconstriction), numbing the area, and discouraging further letdown of milk. Restricting fluids doesn't reduce engorgement and shouldn't be encouraged. Warm compresses will promote blood flow and hence, milk production, worsening the problem of engorgement. Bromocriptine has been removed from the market for lactation suppression.

A client gave birth to a healthy boy 2 days ago. Both mother and baby have had a smooth recovery. The nurse enters the room and tells the client that she and her baby will be discharged home today. The client states, "I do not want to go home." Which of the following is the nurse's most appropriate response? a) Tell the client that she must go home as per hospital policy. b) Ask the client if she has any support in the home. c) Inform the physician that the client does not want to go home. d) Ask the client why she does not want to go home.

Ask the client why she does not want to go home. Correct Explanation: It is important for the nurse to identify the client's concerns and reasons for wanting to stay in the hospital. Open-ended questioning facilitates both effective and therapeutic communication and allows the nurse to address concerns appropriately. Asking about supports at home implies that the nurse has made assumptions about why the client may not want to go home. Informing the physician or telling the client that discharge is hospital policy is not appropriate at this time, because the nurse has not addressed the underlying reason for the client's comment. The client may have safety-related concerns, undisclosed fears, or a need for increased support before discharge. It is imperative that the nurse not make assumptions but further explore concerns.

A woman is bottle-feeding her baby. When the nurse comes into the room the woman says that her breasts are painful and engorged. Which nursing intervention is appropriate? a) Ask if she wants a breast pump to empty her breasts b) Assist the woman in placing ice packs on her breasts c) Assist the woman into the shower and have her run cold water over her breasts d) Explain to the woman that she should breastfeed because she is producing so much milk

Assist the woman in placing ice packs on her breasts Correct Explanation: If the breasts are engorged and the woman is bottle-feeding her newborn, instruct her to keep a support bra on 24 hours per day. Cool compresses or an ice pack wrapped in a towel will usually be soothing and help to suppress milk production.

The nurse's initial action when caring for an infant with a slightly decreased temperature is to a. Notify the physician immediately. b. Place a cap on the infant's head and have the mother perform kangaroo care. c. Tell the mother that the infant must be kept in the nursery and observed for the next 4 hours. d. Change the formula, as this is a sign of formula intolerance.

B

A new mother, who is an adolescent, was cautious at first when holding and touching her newborn. She seemed almost afraid to make contact with baby and only touched it lightly and briefly. However, 48 hours after the birth, the nurse now notices that the new mother is pressing the newborn's cheek against her own and kissing her on the forehead. The nurse recognizes these actions as which of the following? a) Engorgement b) Engrossment c) Involution d) Attachment

Attachment Correct Explanation: When a woman has successfully linked with her newborn it is termed attachment or bonding. Although a woman carried the child inside her for 9 months, she often approaches her newborn not as someone she loves but more as she would approach a stranger. The first time she holds the infant, she may touch only the blanket. Gradually, as a woman holds her child more, she begins to express more warmth, touching the child with the palm of her hand rather than with her fingertips. She smoothes the baby's hair, brushes a cheek, plays with toes, and lets the baby's fingers clasp hers. Soon, she feels comfortable enough to press her cheek against the baby's or kiss the infant's nose; she has successfully bonded or become a mother tending to her child. Engrossment describes the action of new fathers when they stare at their newborn for long intervals. Involution is the process whereby the reproductive organs return to their nonpregnant state. Engorgement is the tension in the breasts as they begin to fill with milk.

A new father wants to know what medication was put into his infant's eyes and why it is needed. The nurse explains to the father that the purpose of the Ilotycin ophthalmic ointment is to a. Destroy an infectious exudate caused by Staphylococcus that could make the infant blind. b. Prevent gonorrheal and chlamydial infection of the infant's eyes potentially acquired from the birth canal. c. Prevent potentially harmful exudate from invading the tear ducts of the infant's eyes, leading to dry eyes. d. Prevent the infant's eyelids from sticking together and help the infant see.

B

To prevent the kidnapping of newborns from the hospital, the nurse should a. Instruct the mother not to give her infant to anyone except the one nurse assigned to her that day. b. Question anyone who is seen walking in the hallways carrying an infant. c. Allow no visitors in the maternity area except those who have identification bracelets. d. Restrict the amount of time infants are out of the nursery.

B

With regard to rubella and Rh issues, nurses should be aware that: A. Breastfeeding mothers cannot be vaccinated with the live attenuated rubella virus. B. Women should be warned that the rubella vaccination is teratogenic and they must avoid pregnancy for 2 to 3 months after vaccination. C. Rh immune globulin is safely administered intravenously because it cannot harm a nursing infant. D. Rh immune globulin boosts the immune system and thereby enhances the effectiveness of vaccinations.

B A. Incorrect: Because the live attenuated rubella virus is not communicable in breast milk, breastfeeding mothers can be vaccinated. B. Correct: Women should understand they must practice contraception for 2 to 3 months after being vaccinated. C. Incorrect: Rh immune globulin is administered intramuscularly (IM); it should never be given to an infant. D. Incorrect: Rh immune globulin suppresses the immune system and therefore might thwart the rubella vaccination. p. 603

What is NOT a reliable indicator of impending shock from early hemorrhage? A. Respirations B. Blood pressure C. Skin condition D. Urinary output

B A. Incorrect: This sign is a more sensitive and more reliable indicator than blood pressure. B. Correct: Blood pressure is not a reliable indicator; several more sensitive signs are available. C. Incorrect: This sign is a more sensitive and more reliable indicator than blood pressure. D. Incorrect: This sign is a more sensitive and more reliable indicator than blood pressure. p. 598

A woman gave birth 48 hours ago to a healthy infant girl. She has decided to bottle feed. During your assessment, you notice that both her breasts are swollen, warm, and tender on palpation. The woman should be advised that this condition can best be treated by: A. Running warm water on her breasts during a shower B. Applying ice to the breasts for comfort C. Expressing small amounts of milk from the breasts to relieve pressure D. Wearing a loose-fitting bra to prevent nipple irritation

B A. Incorrect: This woman is experiencing engorgement, which can be treated by using ice packs (since she is not breastfeeding) and cabbage leaves. B. Correct: This intervention is appropriate for treating engorgement in a mother who is bottle feeding. C. Incorrect: A bottle-feeding mother should avoid any breast stimulation, including pumping or expressing milk. D. Incorrect: A bottle-feeding mother should wear a well-fitted support bra or breast binder continuously for at least the first 72 hours after giving birth. A loose-fitting bra will not aid lactation suppression. Furthermore, the shifting of the bra against the breasts may stimulate the nipples and thereby stimulate lactation. p. 603

What is NOT a postpartum practice for preventing infections? A. Not letting the mother walk barefoot at the hospital B. Using sitz baths and heat lamps more frequently to kill germs C. Having staff members with conditions such as strep throat, conjunctivitis, and diarrhea stay home D. Instructing the mother to change her perineal pad from front to back each time she voids or defecates

B A. Incorrect: Walking barefoot and getting back into bed can contaminate the linens. B. Correct: Heat lamps and sitz baths are now used less frequently because they are hard to keep sterile. C. Incorrect: Staff members with infections need to stay home until they are no longer contagious. D. Incorrect: She should also wash her hands before and after these functions. p. 595

A woman has just delivered a baby. Her prelabor vital signs were T - 98.8 B/P-P-R 120/70, 80, 20. Which combination of findings during the early postpartum period are the most concerning? a) Shaking chills with a fever of 100.3 b) B/P-P-R 90/50, 120, 24 c) Bradycardia and excessive, soaking diaphoresis d) Blood loss of 250 mL and WBC 25,000 cells/mL

B/P-P-R 90/50, 120, 24 Correct Explanation: The decrease in BP with an increase in HR and RR indicate a potential significant complication, and are out of the range of normals, from delivery and need to be reported ASAP. Shaking chills with a temperature of 100.3ºF can occur due to stress on the body and is considered a normal finding. A fever of 100.4ºF should be reported. Options C and D are considered to be within normal limits after delivering a baby.

On assessment of a 2-day postpartum patient the nurses finds the fundus is boggy, at U and slightly to the right. What is the most likely cause of this assessment finding? a) Full bowel b) Uteruine atony c) Bladder distention d) Poor bladder tone

Bladder distention Correct Explanation: The most often cause of a displaced uterus is a distended bladder. Ask the patient to void and then reassess the uterus. According to the scenario described, the most likely cause of the uterine findings would not be uterine atony. A full bowel or poor bladder tone would not cause a boggy and displaced fundus.

A nurse is monitoring the vital signs of a client 24 hours after childbirth. She notes that the client's blood pressure is 100/60 mm Hg. Which of the following postpartum complications should the nurse most suspect in this client, based on this finding? a) Postpartal gestational hypertension b) Bleeding c) Diabetes d) Infection

Bleeding Correct Explanation: Blood pressure should also be monitored carefully during the postpartal period, because a decrease in this can also indicate bleeding. In contrast, an elevation above 140 mm Hg systolic or 90 mm Hg diastolic may indicate the development of postpartal gestational hypertension, an unusual but serious complication of the puerperium. An infection would best be indicated by an elevated oral temperature. Diabetes would be indicated by an elevated blood glucose level.

A yellow crust has formed over the circumcision site. The mother calls the hotline at the local hospital, 5 days after her son was circumcised. She is very concerned. On which rationale should the nurse base her reply? a. After circumcision, the diaper should be changed frequently and fastened snugly. b. This yellow crust is an early sign of infection. c. The yellow crust should not be removed. d. Discontinue the use of petroleum jelly to the tip of the penis.

C

The normal term infant has little difficulty clearing its airway after birth. Most secretions are brought up to the oropharynx by the cough reflex. However, if the infant has excess secretions, the mouth and nasal passages can easily be cleared with a bulb syringe. When instructing parents on the correct use of this piece of equipment, it is important that the nurse teach them to a. Avoid suctioning the nares. b. Insert the compressed bulb into the center of the mouth. c. Suction the mouth first. d. Remove the bulb syringe from the crib when finished.

C

The nurse administers vitamin K to the newborn for what reason? a. Most mothers have a diet deficient in vitamin K, which results in the infant's being deficient. b. Vitamin K prevents the synthesis of prothrombin in the liver and must be given by injection. c. Bacteria that synthesize vitamin K are not present in the newborn's intestinal tract. d. The supply of vitamin K is inadequate for at least 3 to 4 months, and the newborn must be supplemented.

C

When the nurse is in the process of health teaching it is very important that he or she consider the family's cultural beliefs regarding child care. One of these beliefs includes a. Arab women are anxious to breastfeed while still in the hospital. b. It is important to complement Asian parents about their new baby. c. Women from India tie a black thread around the infant's waist. d. In the Korean culture the patient's mother is the primary caregiver of the infant.

C

With regard to lab tests and diagnostic tests in the hospital after birth, nurses should be aware that a. All states test for phenylketonuria (PKU), hypothyroidism, cystic fibrosis, and sickle cell diseases. b. Federal law prohibits newborn genetic testing without parental consent. c. If genetic screening is done before the infant is 24 hours old, it should be repeated at age 1 to 2 weeks. d. Hearing screening is now mandated by federal law

C

A recently delivered mother and her baby are at the clinic for a 6-week postpartum checkup. The nurse should be concerned that psychosocial outcomes are not being met if the woman: A. Discusses her labor and birth experience excessively B. Feels that her baby is more attractive and clever than any others C. Has not given the baby a name D. Partner or family members react very positively about the baby

C A. Incorrect: A new mother who is having difficulty would be unwilling to discuss her labor and birth experience. An appropriate nursing diagnosis might be impaired parenting related to a long, difficult labor or unmet expectations of birth. A mother who is willing to discuss her birth experience is making a healthy personal adjustment. B. Incorrect: The mother who is not coping well would find her baby unattractive and messy. She may also be overly disappointed in the baby's sex. The client might voice concern that the baby reminds her of a family member whom she does not like. C. Correct: If the mother is having difficulty naming her new infant, it may be a signal that she is not adapting well to parenthood. Other red flags include refusal to hold or feed the baby, lack of interaction with the infant, and becoming upset when the baby vomits or needs a diaper change. D. Incorrect: Having a partner and/or other family members react positively is an indication that this new mother has a good support system in place. This support system will help reduce anxiety related to her new role as a mother. pp. 605, 606

If a woman is at risk for thrombus and is not ready to ambulate, nurses might intervene by doing all of these interventions except: A. Putting her in TED hose and/or SCD boots B. Having her flex, extend, and rotate her feet, ankles, and legs C. Having her sit in a chair D. Notifying the physician immediately if a positive Homans' sign occurs

C A. Incorrect: Antiembolic stockings (TED hose) and sequential compression devices (SCD boots) are recommended. Just sitting in a chair will not help. B. Incorrect: Bed exercises such as these are useful. Just sitting in a chair will not help. C. Correct: Sitting immobile in a chair will not help. Bed exercise and prophylactic footwear might. D. Incorrect: A positive Homans' sign (calf muscle pain or warmth, redness, or tenderness) requires the physician's immediate attention. pp. 600-601

Under the Newborns' and Mothers' Health Protection Act, all health plans are required to allow new mothers and newborns to remain in the hospital for a minimum of _____ hours after a normal vaginal birth and for _____ hours after a cesarean birth. A. 24, 73 B. 24, 96 C. 48, 96 D. 48, 120

C A. Incorrect: The specified stays are 48 hours (2 days) for a vaginal birth and 96 hours (4 days) for a cesarean birth. The attending provider and the mother together can decide on an earlier discharge. B. Incorrect: The specified stays are 48 hours (2 days) for a vaginal birth and 96 hours (4 days) for a cesarean birth. The attending provider and the mother together can decide on an earlier discharge. C. Correct: The specified stays are 48 hours (2 days) for a vaginal birth and 96 hours (4 days) for a cesarean birth. The attending provider and the mother together can decide on an earlier discharge. D. Incorrect: The specified stays are 48 hours (2 days) for a vaginal birth and 96 hours (4 days) for a cesarean birth. The attending provider and the mother together can decide on an earlier discharge. p. 589

In providing and teaching cord care, what is an important principle? a. Cord care is done only to control bleeding. b. Alcohol is the only agent used for cord care. c. It takes a minimum of 24 days for the cord to separate. d. The process of keeping the cord dry will decrease bacterial growth.

D

When teaching parents about mandatory newborn screening, it is important for the nurse to explain that the main purpose is to a. Keep the state records updated. b. Allow accurate statistical information. c. Document the number of births. d. Recognize and treat newborn disorders early.

D

A woman gave birth vaginally to a 9-pound, 12-ounce girl yesterday. Her primary health care provider has written orders for perineal ice packs, use of a sitz bath tid, and a stool softener. What information is most closely correlated with these orders? A. The woman is a gravida 2, para 2. B. The woman had a vacuum-assisted birth. C. The woman received epidural anesthesia. D. The woman has an episiotomy.

D A. Incorrect: A multiparous classification is not an indication for these orders. B. Incorrect: A vacuum-assisted birth may be used in conjunction with an episiotomy, which would indicate these interventions. C. Incorrect: Use of epidural anesthesia has no correlation with these orders. D. Correct: These orders are typical interventions for a woman who has had an episiotomy, lacerations, and hemorrhoids. p. 597

The trend in the United States is for women to remain hospitalized no longer than 1 or 2 days after giving birth for all of the following reasons except: A. A wellness orientation rather than a sick-care model B. A desire to reduce health care costs C. Consumer demand for fewer medical interventions and more family-focused experiences D. Less need for nursing time as a result of more medical and technologic advances and devices available at home that can provide information

D A. Incorrect: A wellness orientation seems to focus on getting clients out the door sooner. B. Incorrect: Less hospitalization means lower costs in most cases. C. Incorrect: People believe the family gives more nurturing care than the institution. D. Correct: Nursing time and care are in demand as much as ever; the nurse just has to do things more quickly. p. 585

A 25-year-old multiparous woman gave birth to an infant boy 1 day ago. Today her husband brings a large container of brown seaweed soup to the hospital. When the nurse enters the room, the husband asks for help with warming the soup so that his wife can eat it. The nurse's most appropriate response is to ask the woman: A. "Didn't you like your lunch?" B. "Does your doctor know that you are planning to eat that?" C. "What is that anyway?" D. "I'll warm the soup in the microwave for you."

D A. Incorrect: Cultural dietary preferences must be respected. B. Incorrect: Women may request that family members bring favorite or culturally appropriated foods to the hospital. C. Incorrect: Cultural dietary preferences must be respected. A statement such as this does not show cultural sensitivity. D. Correct: This statement shows cultural sensitivity to the dietary preferences of the woman and is the most appropriate response. p. 601

In a variation of rooming-in, called couplet care, the mother and infant share a room and the mother shares the care of the infant with: A. The father of the infant B. Her mother (the infant's grandmother) C. Her eldest daughter (the infant's sister) D. The nurse

D A. Incorrect: In couplet care, the mother shares a room with the newborn and shares infant care with a nurse educated in maternity and infant care. B. Incorrect: In couplet care, the mother shares a room with the newborn and shares infant care with a nurse educated in maternity and infant care. C. Incorrect: In couplet care, the mother shares a room with the newborn and shares infant care with a nurse educated in maternity and infant care. D. Correct: In couplet care, the mother shares a room with the newborn and shares infant care with a nurse educated in maternity and infant care. p. 594

Nursing care in the fourth trimester includes an important intervention sometimes referred to as taking the time to mother the mother. Specifically, this expression refers to: A. Formally initializing individualized care by confirming the woman's and infant's ID numbers on their respective wrist bands ("This is your baby.") B. Teaching the mother to check the identity of any person who comes to remove the baby from the room ("It's a dangerous world out there.") C. Including other family members in the teaching of self-care and child care ("We're all in this together.") D. Nurturing the woman by providing encouragement and support as she takes on the many tasks of motherhood

D A. Incorrect: Matching ID wrist bands is more of a formality but also a get-acquainted procedure. "Mothering the mother" is more a process of encouraging and supporting the woman in her new role. B. Incorrect: Having the mother check IDs is a security measure for protecting the baby from abduction. "Mothering the mother" is more a process of encouraging and supporting the woman in her new role. C. Incorrect: Teaching the whole family is just good nursing practice. "Mothering the mother" is more a process of encouraging and supporting the woman in her new role. D. Correct: Many professionals believe that the nurse's nurturing and support function is more important than providing physical care and teaching. p. 595

On examining a woman who gave birth 5 hours ago, the nurse finds that the woman has completely saturated a perineal pad within 15 minutes. The nurse's first action is to: A. Begin an intravenous (IV) infusion of Ringer's lactate solution B. Assess the woman's vital signs C. Call the woman's primary health care provider D. Massage the woman's fundus

D A. Incorrect: The nurse may begin an IV infusion to restore circulatory volume, but this would not be the first action. B. Incorrect: Blood pressure is not a reliable indicator of impending shock from impending hemorrhage; assessing vital signs should not be the nurse's first action. C. Incorrect: The physician would be notified after the nurse completes the assessment of the woman. D. Correct: The nurse should assess the uterus for atony. Uterine tone must be established to prevent excessive blood loss. p. 598

Which finding could prevent early discharge of a newborn who is now 12 hours old? A. Birth weight of 3000 g B. One meconium stool since birth C. Voided, clear, pale urine three times since birth D. Infant breastfed once with some difficulty latching on and sucking and once with some success for about 5 minutes on each breast.

D A. Incorrect: This is a normal infant finding and would not prevent early discharge. B. Incorrect: This is a normal infant finding and would not prevent early discharge. C. Incorrect: This is a normal infant finding and would not prevent early discharge. D. Correct: This finding indicates that the infant is having some difficulty with breastfeeding. The infant needs to complete at least two successful feedings (normal sucking and swallowing) before an early discharge. p. 590

When caring for a postpartum client who has given birth vaginally, the nurse assesses the client's respiratory status, noting that it has quickly returned to normal. The nurse understands that which of the following is responsible for this change? a) Decreased bladder pressure b) Increased progesterone levels c) Decreased intra-abdominal pressure d) Use of anesthesia during delivery

Decreased intra-abdominal pressure Correct Explanation: The nurse should identify decreased intra-abdominal pressure as the cause of the respiratory system functioning normally. Progesterone levels do not influence the respiratory system. Decreased bladder pressure does not affect breathing. Anesthesia used during delivery causes the respiratory system to take a longer time to return to normal.

For several hours after delivery, Norah, a multigravida who experienced a much more difficult labor this time than any time previously, wants to talk about why the birthing process was so hard for her this time. In fact, she's focusing on this aspect to the point that she seems relatively indifferent to her newborn. How should you handle this situation? a) Redirect her attention to the baby by reminding her of the details of newborn care b) Encourage her to discuss her experience of the birth and answer any questions or concerns she may have c) Point out positive features of her baby and encourage her to hold and cuddle the baby d) Ask her to describe how she plans to integrate the newcomer into her existing family, including any actions she has taken to prepare the siblings

Encourage her to discuss her experience of the birth and answer any questions or concerns she may have Explanation: The patient needs to explore her birth experience and clarify her questions. The nurse should allow her to ask questions, be supportive and encourage her to express her feelings. Redirecting her attention to the baby, asking her to describe how she plans to integrate the new baby into the family, or pointing out positive features of the new baby do not meet the needs of the patient at this time, and are therefore incorrect answers.

The patient under your care is complaining she has not had a bowel moment since her infant was born 2 days ago. She asks the nurse what she can do to help her have a bowel movement. What intervention is appropriate to encourage having a bowel movement? 1. Add dairy products to her diet 2. Give a stimulant laxative 3. Encourage fiber rich foods 4. Holding the feces until there is a strong urge to defecate.

Encourage the patient to eat more fiber rich foods Correct Explanation: Encouraging fiber rich foods will help with prevention of constipation. The patient needs plenty of water, to ambulate, and take stool softeners if ordered by the provider. Offering a stimulant laxative is not appropriate. Adding dairy products to the diet may be a good thing, but will not generally produce a bowel movement. Holding the feces until there is a strong urge to defecate will only increase the risk of constipation as well as possible resultant complications.

If a mother and her family have freely chosen early discharge from the hospital, the nurse and the health care provider are not legally responsible if complications occur and her condition had not been stabilized within normal limits. True or false?

False Even if the mother chose to leave, the medical and nursing staffs still could be sued for abandonment. p. 590

A woman who has just given birth seems to be bonding with her newborn, despite the fact that earlier in labor she had expressed an intent to give the baby up for adoption. In this case, the nurse should encourage the mother to keep her baby. a) True b) False

False Correct Explanation: Do not attempt to change a woman's mind about keeping her child or placing the child for adoption during the postpartal period as she is extremely vulnerable to suggestion at this time, and such decisions are too long range and too important to be made at such an emotional time. Her earlier conclusion may be the sound one. Instead, offer nonjudgmental support. Be especially aware of your own feelings about this issue, to avoid influencing a woman's decision making unnecessarily.

A nurse is caring for a breastfeeding client who complains of engorgement. The nurse identifies that the client's condition is due to not fully emptying her breasts at each feeding. Which of the following should the nurse suggest to help her prevent engorgement? a) Feed the baby at least every two or three hours b) Apply cold compresses to the breasts c) Dry the nipples following feedings d) Provide the infant oral nystatin

Feed the baby at least every two or three hours Correct Explanation: The nurse should suggest the client feed the baby every two or three hours to help her reduce and prevent further engorgement. Application of cold compresses to the breasts is suggested to reduce engorgement for non-breastfeeding clients. If the mother has developed a candidal infection on the nipples, the treatment involves application of an antifungal cream to the nipples following feedings and providing the infant with oral nystatin. The nurse can suggest drying the nipples following feedings if the client experiences nipple pain.

When assessing a postpartum woman, which finding would be most significant in identifying possible postpartum hemorrhage? a) Increased cardiac output b) Increase heart rate c) Increased hematocrit level d) Increased blood pressure

Increase heart rate Correct Explanation: Tachycardia in the postpartum woman warrants further investigation. Typically the postpartum woman is bradycardic for the first 2 weeks. In most instances of postpartum hemorrhage, blood pressure and cardiac output remain increased because of a compensatory increase in heart rate. Red blood cell production ceases early in the puerperium, causing hemoglobin and hematocrit levels to decrease slightly in the first 24 hours and then rise slowly. Hematocrit would be unreliable as an indicator of hemorrhage.

The process by which the reproductive organs return to the nonpregnant size and function is termed what? a) Evolution b) Involution c) Decrement d) Progression

Involution Correct Explanation: Involution is the term used to describe the process of the return to nonpregnancy size and function of reproductive organs. Evolution is change in the genetic material of a population of organisms from one generation to the next. Decrement is the act or process of decreasing . Progression is defined as movement through stages such as the progression of labor. Options A, C, and D are distracters for this question.

Healthy bonding behaviors are important to note when you are assessing the new family. What would you consider a warning sign that the mother and infant were not attaching as they should? a) Mother wants you in the room while she breastfeeds as she is afraid she isn't doing it right. b) Mother states she is concerned about one of her other children not liking the new baby. c) Mother cries and says "I have no family nearby and my mother-in-law doesn't like me." d) Mother states she wanted a boy this time, not another girl.

Mother states she wanted a boy this time, not another girl. Correct Explanation: It is important to differentiate between a new parent who is nervous and anxious about her new role and one who is rejecting her parenting role. Warning signals of poor attachment include turning away from the newborn, refusing or neglecting to provide care, and disengagement from the newborn.

A nurse is caring for a client who is nursing her baby boy. The client complains of afterpains. Secretion of which of the following should the nurse identify as the cause of afterpains? a) Oxytocin b) Prolactin c) Estrogen d) Progesterone

Oxytocin Correct Explanation: Secretion of oxytocin stimulates uterine contraction and causes the woman to experience afterpains. Decrease in progesterone and estrogen after placental delivery stimulates the anterior pituitary to secrete prolactin which causes lactation

A postpartum client complains of stress incontinence. What information should the nurse suggest to the client to overcome stress incontinence? a) Perform aerobic exercises b) Frequently empty the bladder c) Reduce fluid intake d) Perform Kegel's exercises

Perform Kegel's exercises Correct Explanation: The nurse should ask the client to perform the Kegel's exercises in which the client needs to alternately contract and relax the perineal muscles. Aerobic exercises will not help to strengthen perineal muscles. Reduced fluid intake and frequent emptying of the bladder will not help the client overcome stress incontinence.

You are used to working on the postpartum floor taking care of women who have had normal vaginal deliveries. Today, however, you have been assigned to help care for woman who are less than 24 hours post cesarean delivery. You know that in making your assessments you will have to change some things that you would not normally assess. What would you leave out of your patient assessments? a) Respiratory status b) Lower extremities c) Perineum d) Breasts

Perineum Correct Explanation: Usually a woman who experiences cesarean birth does not have an episiotomy, although rarely this may be the case.

During a postpartum exam on the day of delivery, the woman complains that she is still so sore that she can't sit comfortably. You examine her perineum and find the edges of the episiotomy approximated without signs of a hematoma. Which intervention will be most beneficial at this point? a) Notify a physician b) Apply a warm washcloth c) Place an ice pack d) Put on a witch hazel pad.

Place an ice pack Correct Explanation: The labia and perineum may be edematous after delivery and bruised, the use of ice would assist in decreasing the pain and swelling. Applying a warm washcloth would bring more blood as well as fluid to the sore area, thereby increasing the edema and the soreness. Applying a witch hazel pad needs the order of the physician. Notifying a physician is not necessary at this time as this is considered a normal finding.

A woman who delivered a healthy baby 5 days ago is experiencing fatigue and weepiness, lasting for short periods each day. Which of the following factors/conditions does the nurse believe is causing this experience? a) Postpartum depression. b) Postpartum reaction. c) Postpartum anxiety. d) Postpartum baby blues.

Postpartum baby blues. Explanation: Postpartum baby blues is common in women after giving birth. It is a mild depression; however, functioning usually is not impaired. Postpartum blues usually peaks at day 4 or 5 after birth. Postpartum anxiety and postpartum depression do not usually start until at least 3 to 4 weeks and up to 1 year following the birth of a baby. Postpartum reaction is a term to include postpartum depression, anxiety, and psychosis.

A nurse is caring for a client on the second day postpartum. The client informs the nurse that she is voiding a large volume of urine frequently. Which of the following should the nurse identify as a potential cause for urinary frequency? a) Urinary overflow b) Postpartum diuresis c) Trauma to pelvic muscles d) Urinary tract infection

Postpartum diuresis Correct Explanation: The nurse should identify postpartum diuresis as the potential cause for urinary frequency. Urinary overflow occurs if the bladder is not completely emptied. Urinary tract infection may be accompanied by fever and a burning sensation. Trauma to pelvic muscles does not affect urinary frequency.

What is the primary function of uterine contractions after delivery of the infant and placenta? a) Return the uterus to normal size b) Seal off the blood vessels at the site of the placenta c) Stop the flow of blood d) Close the cervix

Seal off the blood vessels at the site of the placenta Correct Explanation: The contractions of the uterus help to constrict the vessels where the placenta was located. This does decrease the flow of blood, but is secondary in occurrence to the constriction of the blood vessels. Uterine contraction also leads to uterine involution, which normally occurs at a predictable rate. Uterine involution assists in closing the cervix. Again, options A, C, and D are secondary to the constriction of blood vessels at the placental site.

Which maternal reaction is the most concerning? a) She is tearful for several days and has difficulty eating and sleeping b) She hesitates to take her newborn when offered and expresses disappointment with the way the baby looks c) She expresses doubt about her ability to care for the baby as well as the nurse can d) She neglects to engage with or provide care for the baby and shows little interest in it

She neglects to engage with or provide care for the baby and shows little interest in it Correct Explanation: A mother not bonding with the infant or showing disinterest is a cause for concern and requires a referral or notification of the primary health care provider. Some mothers hesitate to take their newborn, and express disappointment in the way the baby looks, especially if they want a child of one sex and have a child of the opposite sex. Expressing doubt about the ability to care for the baby is not unusual, and being tearful for several days with difficulty eating and sleeping is common with "postpartum blues".

A nurse is caring for a client in the postpartum period. The nurse observes that distention of the abdominal muscles during pregnancy has resulted in separation of the rectus muscles. What intervention should the nurse perform to assist in healing the distended abdominal muscles? a) Applying warm compresses b) Massaging the muscles c) Applying moist heat d) Suggesting proper exercise

Suggesting proper exercise Correct Explanation: The nurse should suggest proper exercise to the client to heal the distended abdominal muscles. Application of warm compresses, application of moist heat, and massaging the muscles gently are not suggested for distended abdominal muscles.

A new mother gave birth to her baby 24 hours ago and today has been content to rest in her hospital bed, hold her baby, allow the nurse to care for her, and to discuss her labor and birth experience with visitors. Which phase of the puerperium is this client in? a) Taking-hold phase b) Taking-in phase c) Letting-go phase d) Rooming-in phase

Taking-in phase Correct Explanation: The taking-in phase is largely a time of reflection. During this 1- to 3-day period, a woman is largely passive. She prefers having a nurse attend to her needs and make decisions for her, rather than do these things herself. As a part of thinking and pondering about her new role, the woman usually wants to talk about her pregnancy, especially about her labor and birth. After a time of passive dependence, a woman enters the taking-hold phase and begins to initiate action. She prefers to get her own washcloth or to make her own decisions. In the letting-go phase, a woman finally redefines her new role. She gives up the fantasized image of her child and accepts the real one; she gives up her old role of being childless or the mother of only one or two (or however many children she had before this birth). Rooming-in is a feature offered by hospitals in which the infant is allowed to stay in the same hospital room as the mother following birth; it is not a phase of the puerperium.

Rubin identified a series of changes that a new mother makes during the postpartum period. The correct sequence of these changes is a) Taking-in, taking-hold, letting-go b) Taking, holding-on, letting-go c) Taking-in, holding-on, letting-go d) Taking-in, taking-on, letting-go

Taking-in, taking-hold, letting-go Correct Explanation: The new mother makes progressive changes to know her infant, review the pregnancy and labor, validate her safe passage through these phases, learn the initial tasks of mothering, and let go of her former life to incorporate this new child.

While educating a class of postpartum patients before discharge home after delivery, one woman asks when "will I stop bleeding?" How should the nurse respond? a) Bleeding may occur on and off for the next 2 to 3 weeks b) You should stop bleeding and have no discharge in the next 1 to 2 weeks c) The bleeding may slowly decrease over the next 1 to 3 weeks, changing color to a white discharge, which may continue for up to 6 weeks d) The bleeding may continue for 6 weeks

The bleeding may slowly decrease over the next 1 to 3 weeks, changing color to a white discharge, which may continue for up to 6 weeks Correct Explanation: The lochia changes color in the first few weeks postpartum; the active bleeding stops in the first week but a white discharge may continue for up to 6 weeks after delivery. Option A is incorrect because it is an incomplete answer. Option B is incorrect because bleeding does not occur "off and on"; the bleeding stops during the first week but a discharge continues to occur. Option C is incorrect because the discharge may continue for up to six weeks

Which reason explains why women should be encouraged to perform Kegel exercises after delivery? a) They promote blood flow, enabling healing and muscle strengthening. b) They promote the return of normal bowel function. c) They assist the woman in burning calories for rapid postpartum weight loss. d) They assist with lochia removal.

They promote blood flow, enabling healing and muscle strengthening. Correct Explanation: Exercising the pubococcygeal muscle increases blood flow to the area. The increased blood flow brings oxygen and other nutrients to the perineal area to aid in healing. Additionally, these exercises help strengthen the musculature, thereby decreasing the risk of future complications, such as incontinence and uterine prolapse. Performing Kegel exercises may assist with lochia removal, but that isn't their main purpose. Bowel function isn't influenced by Kegel exercises. Kegel exercises don't generate sufficient energy expenditure to burn many calories.

An important nursing intervention is maintaining safe glucose levels in the newborn. A common practice is to feed infants either breast milk or formula if glucose screening shows results of 40 to 45 mg/dL or less. Is this statement true or false?

True

Regardless of her obstetric status, no woman should be discharged from the recovery area until she has completely recovered from the effects of anesthesia and has been cleared by a member of the anesthesia care team. True or false?

True It takes several hours to recover from anesthesia. Obstetric recovery areas are held to the same standard of care expected for any postanesthesia recovery. p. 587

The nurse is caring for a client in the postpartum period. The client has difficulty in voiding and is catheterized. The nurse then would monitor the client for which of the following? a) Loss of pelvic muscle tone b) Stress incontinence c) Urinary tract infection d) Increased urine output

Urinary tract infection Correct Explanation: The nurse would need to monitor the client for signs and symptoms of a urinary tract infection, a risk associated with catheterization. Stress incontinence is caused due to loss of pelvic muscle tone after birth. Increased urinary output is observed in diuresis. Catheterization does not cause loss of pelvic muscle tone, increased urine output, or stress incontinence.

The nurse is making a home visit to a woman who is 4 days postpartum. Which finding would indicate to the nurse that the woman is experiencing a problem? a) Edematous vagina b) Lochia serosa c) Diaphoresis d) Uterus 1 cm below umbilicus

Uterus 1 cm below umbilicus Correct Explanation: By the fourth postpartum day, the uterus should be approximately 4 cm below the umbilicus. Being only at 1 cm indicates that the uterus is not contracting as it should. Lochia serosa is normal from days 3 to 10 postpartum. After birth the vagina is edematous and thin with few rugae. It eventually thickens and rugae return in approximately 3 weeks. Diaphoresis is common during the early postpartum period, especially in the first week. It is a mechanism to reduce fluids retained during pregnancy and restore prepregnant body fluid levels.

The nurse is providing education to a mother who is going to bottle feed her infant. What information will the nurse provide to this mom regarding breast care? a) Wear a tight, supportive bra b) Massage the breast when they are painful c) Express small amounts of milk when they are too full d) Run warm water over the breast in the shower

Wear a tight, supportive bra Explanation: The patient trying to dry up her milk supply should do as little stimulation to the breast as possible. She needs to wear a tight, supportive bra and use ice. Running warm water over the breasts in the shower will only stimulate the secretion, and therefore the production, of milk. Massaging the breasts will stimulate them to expel the milk and therefore produce more milk, as will expressing small amounts of milk when the breasts are full.

A nurse is caring for a non-breastfeeding client in the postpartum period. The client complains of engorgement. What suggestion should the nurse provide to alleviate breast discomfort? a) Express milk frequently b) Apply hydrogel dressing c) Wear a well-fitting bra d) Apply warm compress

Wear a well-fitting bra Correct Explanation: The nurse should suggest the client wear a well-fitting bra to provide support and help alleviate breast discomfort. Application of warm compress and expressing milk frequently is suggested to alleviate breast engorgement in breastfeeding clients. Hydrogel dressings are used prophylactically in treating nipple pain.

During the immediate postpartum period, saturation of one pad within 2 hours is considered ____________________ blood loss.

heavy p. 597


Conjuntos de estudio relacionados

total quality management (TQM) Chapter 5

View Set

Chapter 11 International Marketing

View Set

Module 17: Basic Concepts of Sensation & Perception

View Set

Teacher & training semester 1 study guide

View Set

Personal finance chapter 3 short answer

View Set